You are on page 1of 91

Part 2 : 11/10/17 09:52:04

Question 1 - CIA 593 IV.40 - Ratios: Liquidity, Leverage, Coverage and Activity

A condensed comparative balance sheet for a company appears below:


12-31-Year 1 12-31-Year 2
Cash $ 40,000 $ 30,000
Accounts receivable 120,000 100,000
Inventory 200,000 300,000
Property, plant & equipment 500,000 550,000
Accumulated depreciation (280,000) (340,000)
Total assets $580,000 $640,000
Current liabilities $60,000 $100,000
Long-term liabilities 390,000 420,000
Stockholders' equity 130,000 120,000
Total liabilities and equity $580,000 $640,000

In looking at liquidity ratios at both balance sheet dates, what happened to the (1) current ratio and (2) acid-test (quick)
ratio?

A. (1) Decreased (2) Increased


B. (1) Increased (2) Decreased
C. (1) Increased (2) Increased
D. (1) Decreased (2) Decreased

Question 2 - ICMA 10.P2.037 - Ratios: Liquidity, Leverage, Coverage and Activity

The capital structure of four corporations is as follows.


Corporation
Sterling Cooper Warwick Pane
Short-term debt 10% 10% 15% 10%
Long-term debt 40% 35% 30% 30%
Preferred stock 30% 30% 30% 30%
Common equity 20% 25% 25% 30%

Which corporation is the most highly leveraged?

A. Cooper.
B. Pane.
C. Sterling.
D. Warwick.

Question 3 - ICMA 10.P2.035 - Ratios: Liquidity, Leverage, Coverage and Activity

Dedham Corporation has decided to include certain financial ratios in its year-end annual report to shareholders.
Selected information relating to its most recent fiscal year is provided below.
Cash $ 10,000
Accounts receivable 20,000
Prepaid expenses 8,000
Inventory 30,000
Available-for-sale securities classified as current assets
At cost 9,000

(c) HOCK international, page 1


Part 2 : 11/10/17 09:52:04

Fair value at year end 12,000


Accounts payable 15,000
Notes payable (due in 90 days) 25,000
Bonds payable (due in 10 years) 35,000

Dedham's quick (acid-test) ratio at year end is

A. 1.80 to 1.
B. 2.00 to 1.
C. 1.05 to 1.
D. 1.925 to 1.

Question 4 - CMA 692 1.9 - Ratios: Liquidity, Leverage, Coverage and Activity

Carlisle Company currently sells 400,000 bottles of perfume each year. Each bottle costs $0.84 to produce and sells
for $1.00. Fixed costs are $28,000 per year. The firm has annual interest expense of $6,000, preferred stock dividends
of $2,000 per year, and a 40% tax rate. Carlisle uses the following formulas to determine the company's leverage.

Operating leverage = [Q(S − VC)] ÷ [Q(S − VC) − FC]

Financial leverage = EBIT ÷ {EBIT − I − [P / (1 − T)]}

Total leverage = Q(S − VC) ÷ {Q(S − VC) − FC − I − [P / (1 − T)]}

Where:
Q=Quantity
FC=Fixed Cost
VC=Variable Cost
S=Selling Price
I=Interest Expense
P=Preferred Dividends
T=Tax Rate
EBIT=Earnings Before Interest and Taxes

The degree of financial leverage for Carlisle Company is

A. 2.3
B. 1.35
C. 2.4
D. 1.78

Question 5 - ICMA 10.P2.040 - Ratios: Liquidity, Leverage, Coverage and Activity

Firms with high degrees of financial leverage would be best characterized as having

A. high debt-to-equity ratios.


B. zero coupon bonds in their capital structures.
C. low current ratios.
D. high fixed-charge coverage.

(c) HOCK international, page 2


Part 2 : 11/10/17 09:52:04

Question 6 - CMA 695 2.2 - Ratios: Liquidity, Leverage, Coverage and Activity

CPZ Enterprises had the following account information.


Accounts receivable $200,000
Accounts payable 80,000
Bonds payable, due in ten years 10,000
Cash 100,000
Interest payable, due in three months 10,000
Inventory 400,000
Land 250,000
Notes payable, due in six months 50,000
Prepaid expenses 40,000

The company has an operating cycle of five months.

What is the company's acid test (quick) ratio?

A. 2.14
B. 1.68
C. 0.68
D. 2.31

Question 7 - CMA 693 2.2 - Ratios: Liquidity, Leverage, Coverage and Activity

Lisa, Inc.
Statement of Financial Position
December 31, 20X4
(in thousands)
20X4 20X3
Assets
Current assets:
Cash $ 30 $ 25
Trading securities 20 15
Accounts receivable (net) 45 30
Inventories (at lower of cost of market) 60 50
Prepaid items 15 20
Total Current Assets $170 $140

Long-term assets:
Long-term investments:
Available-for-sale investments $ 25 $ 20
Property, plant & equipment:
Land (at cost) 75 75
Building (net) 80 90
Equipment (net) 95 100
Intangible assets:
Patents (net) 35 17
Goodwill (net) 20 13
Total Long-Term Assets $330 $315
Total Assets $500 $455

(c) HOCK international, page 3


Part 2 : 11/10/17 09:52:04

Liabilities and Equity


Current liabilities:
Notes payable $ 23 $ 12
Accounts payable 47 28
Accrued interest $ 15 $ 15
Total current liabilities $ 85 $ 55

Long-term liabilities:
Long-term Notes payable 10% due 12/31/20X6 $ 10 $ 10
Bonds payable 12% due 12/31/20X9 15 15
Total long-term debt $ 25 $ 25
Total liabilities $110 $ 80

Shareholders' Equity
Preferred stock - 5% cumulative, $100 par, nonparticipating
authorized, issued and outstanding, 1,000 shares $100 $100
Common stock - $10 par 20,000 shares authorized, 15,000
shares issued and outstanding 150 150
Additional paid-in capital - common 75 75
Retained earnings 65 50
Total Equity $390 $375
Total Liabilities & Equity $500 $455

Assume net credit sales and cost of goods sold for 20X4 were $300,000 and $220,000 respectively. Lisa Inc.'s
accounts receivable turnover for 20X4 was

A. 5.9 times.
B. 8.0 times.
C. 4.9 times.
D. 6.7 times.

Question 8 - ICMA 10.P2.060 - Ratios: Liquidity, Leverage, Coverage and Activity

Cornwall Corporation's net accounts receivable were $68,000 and $47,000 at the beginning and end of the year,
respectively. Cornwall's condensed Income Statement is shown below.
Sales $900,000
Cost of goods sold 527,000
Operating expenses 175,000
Operating income 198,000
Income tax 79,000
Net income $119,000

Cornwall's average number of days' sales in accounts receivable (using a 360-day year) is

A. 8 days.
B. 13 days.
C. 19 days.
D. 23 days.

Question 9 - CMA 1294 2.23 - Ratios: Liquidity, Leverage, Coverage and Activity

(c) HOCK international, page 4


Part 2 : 11/10/17 09:52:04

The following inventory and sales data are available for the current year for Volpone Company. Volpone uses a
365-day year when computing ratios.
November 30, 2010 November 30, 2009
Net credit sales $6,205,000
Gross receivables 350,000 320,000
Inventory 960,000 780,000
Cost of goods sold 4,380,000

Volpone Company's average number of days to sell inventory for the current year is

A. 72.50 days.
B. 80.00 days.
C. 51.18 days.
D. 65.00 days.

Question 10 - ICMA 10.P2.053 - Ratios: Liquidity, Leverage, Coverage and Activity

Maydale Inc.'s financial statements show the following information.


Accounts receivable, end of Year 1$ 320,000
Credit sales for Year 2 3,600,000
Accounts receivable, end of Year 2 400,000

Maydale's accounts receivable turnover ratio is

A. 10.00.
B. 11.25.
C. 0.10.
D. 9.00.

Question 11 - ICMA 10.P2.026 - Ratios: Liquidity, Leverage, Coverage and Activity

Markowitz Company increased its allowance for uncollectible accounts. This adjustment will

A. reduce the current ratio.


B. reduce debt-to-asset ratio.
C. increase the acid test ratio.
D. increase working capital.

Question 12 - ICMA 10.P2.042 - Ratios: Liquidity, Leverage, Coverage and Activity

A financial analyst with Mineral Inc. calculated the company's degree of financial leverage as 1.5. If net income before
interest increases by 5%, earnings to shareholders will increase by

A. 7.50%.
B. 5.00%.
C. 1.50%.
D. 3.33%.

(c) HOCK international, page 5


Part 2 : 11/10/17 09:52:04

Question 13 - ICMA 10.P2.049 - Ratios: Liquidity, Leverage, Coverage and Activity

The interest expense for a company is equal to its earnings before interest and taxes (EBIT). The company's tax rate is
40%. The company's times-interest earned ratio is equal to

A. 1.2.
B. 0.6.
C. 2.0.
D. 1.0.

Question 14 - ICMA 10.P2.054 - Ratios: Liquidity, Leverage, Coverage and Activity

Zubin Corporation experiences a decrease in sales and the cost of good sold, an increase in accounts receivable, and
no change in inventory. If all else is held constant, what is the total effect of these changes on the receivables turnover
and inventory ratios?

A. Inventory turnover decreased; receivables turnover increased.


B. Inventory turnover increased; receivables turnover increased.
C. Inventory turnover decreased; receivables turnover decreased.
D. Inventory turnover increased; receivables turnover decreased.

Question 15 - ICMA 10.P2.062 - Ratios: Liquidity, Leverage, Coverage and Activity

On its year-end financial statements, Caper Corporation showed sales of $3,000,000, net fixed assets of $1,300,000,
and total assets of $2,000,000. The company's fixed asset turnover is

A. 1.5 times.
B. 43.3%.
C. 66.7%.
D. 2.3 times.

Question 16 - CMA 690 1.16 - Ratios: Liquidity, Leverage, Coverage and Activity

This year, Nelson Industries increased earnings before interest and taxes (EBIT) by 17%. During the same period, net
income after tax increased by 42%. The degree of financial leverage that existed during the year is:

A. 2.47.

What is the answer if EBIT decreases by 17 %


B. 5.90.
C. 4.20.
D. 1.70.

Question 17 - CIA 590 IV.47 - Ratios: Liquidity, Leverage, Coverage and Activity

Given an acid test ratio of 2.0, current assets of $5,000, and inventory of $2,000, the value of current liabilities is

(c) HOCK international, page 6


Part 2 : 11/10/17 09:52:04

A. $3,500
B. $2,500
C. $6,000
D. $1,500

Question 18 - CMA 692 1.8 - Ratios: Liquidity, Leverage, Coverage and Activity

Carlisle Company currently sells 400,000 bottles of perfume each year. Each bottle costs $0.84 to produce and sells
for $1.00. Fixed costs are $28,000 per year. The firm has annual interest expense of $6,000, preferred stock dividends
of $2,000 per year, and a 40% tax rate. Carlisle uses the following formulas to determine the company's leverage.

Operating leverage = [Q(S − VC)] ÷ [Q(S − VC) − FC]

Financial leverage = EBIT ÷ {EBIT − I − [P / (1 − T)]}

Total leverage = Q(S − VC) ÷ {Q(S − VC) − FC − I − [P / (1 − T)]}

Where:
Q=Quantity
FC=Fixed Cost
VC=Variable Cost
S=Selling Price
I=Interest Expense
P=Preferred Dividends
T=Tax Rate
EBIT=Earnings Before Interest and Taxes

The degree of operating leverage for Carlisle Company is

A. 1.35
B. 1.78
C. 1.2
D. 2.4

Question 19 - ICMA 10.P2.057 - Ratios: Liquidity, Leverage, Coverage and Activity

Makay Corporation has decided to include certain financial ratios in its year-end annual report to shareholders.
Selected information relating to its most recent fiscal year is provided below.
Cash $ 10,000
Accounts receivable (end of year) 20,000
Accounts receivable (beginning of year) 24,000
Inventory (end of year) 30,000
Inventory (beginning of year) 26,000
Notes payable (due in 90 days) 25,000
Bonds payable (due in 10 years) 35,000
Net credit sales for year 220,000
Cost of goods sold 140,000

Makay's average inventory turnover for the year was

A. 4.7 times.

(c) HOCK international, page 7


Part 2 : 11/10/17 09:52:04

B. 5.0 times.
C. 5.4 times.
D. 7.9 times.

Question 20 - CMA 1295 2.13 - Ratios: Liquidity, Leverage, Coverage and Activity

All of the following financial indicators are measures of either liquidity or activity except the

A. Accounts receivable turnover.


B. Average collection period in days.
C. Merchandise inventory turnover.
D. Times-interest-earned ratio.

Question 21 - CMA 1293 2.17 - Ratios: Liquidity, Leverage, Coverage and Activity

Norton, Inc. has a 2-to-1 current ratio. This ratio would increase to more than 2 to 1 if

A. The company wrote off an uncollectible receivable.


B. A previously declared stock dividend was distributed.
C. The company purchased inventory on open account.
D. The company sold merchandise on open account that earned a normal gross margin.

Question 22 - CIA 595 IV.51 - Ratios: Liquidity, Leverage, Coverage and Activity

Everything else being equal, a <<_____>> highly leveraged firm will have <<_____>> earnings per share.

A. Less; Less volatile


B. More; Less volatile
C. Less; Higher
D. More; Lower

Question 23 - CMA 688 4.15 - Ratios: Liquidity, Leverage, Coverage and Activity

The days' sales in receivables ratio will be understated if the company

A. Uses a calendar year for its accounting period.


B. Does not use average receivables in the ratio calculation.
C. Uses a natural business year for its accounting period.
D. Uses average receivables in the ratio calculation.

Question 24 - ICMA 10.P2.025 - Ratios: Liquidity, Leverage, Coverage and Activity

Davis Retail Inc. has total assets of $7,500,000 and a current ratio of 2.3 times before purchasing $750,000 of

(c) HOCK international, page 8


Part 2 : 11/10/17 09:52:04

merchandise on credit for resale. After this purchase, the current ratio will

A. remain at 2.3 times.


B. be lower than 2.3 times.
C. be higher than 2.3 times.
D. be exactly 2.53 times.

Question 25 - CMA 687 4.27 - Ratios: Liquidity, Leverage, Coverage and Activity

When compared to a debt-to-assets ratio, a debt to equity ratio would

A. Have no relationship at all to the debt to assets ratio.


B. Be about the same as the debt to assets ratio.
C. Be higher than the debt to assets ratio.
D. Be lower than the debt to assets ratio.

Question 26 - ICMA 10.P2.021 - Ratios: Liquidity, Leverage, Coverage and Activity

Broomall Corporation has decided to include certain financial ratios in its year-end annual report to shareholders.
Selected information relating to its most recent fiscal year is provided below.
Cash $ 10,000
Accounts receivable 20,000
Prepaid expenses 8,000
Inventory 30,000
Available-for-sale securities classified as current assets
At cost 9,000
Fair value at year end 12,000
Accounts payable 15,000
Notes payable (due in 90 days) 25,000
Bonds payable (due in 10 years) 35,000
Net credit sales for year 220,000
Cost of goods sold 140,000

Broomall's working capital at year end is

A. $37,000.
B. $40,000.
C. $10,000.
D. $28,000.

Question 27 - ICMA 13.P2.004 - Ratios: Liquidity, Leverage, Coverage and Activity

Financial information for Arbat Inc. for two years of operation is shown below.
Year 1 Year 2
Sales $4,000,000$4,400,000
Total operating costs 3,200,000 3,440,000
Earnings before interest and taxes $ 800,000 $ 960,000
Interest payments 320,000 275,000

(c) HOCK international, page 9


Part 2 : 11/10/17 09:52:04

Income taxes 245,000 354,000


Net income $ 235,000 $ 331,000

Earnings per share $ 2.35 $ 3.31

The degree of operating leverage for Arbat Inc. is

A. 2.67.
B. 2.00.
C. 0.75.
D. 4.09.

Question 28 - ICMA 10.P2.044 - Ratios: Liquidity, Leverage, Coverage and Activity

The Liabilities and Shareholders' Equity section of Mica Corporation's Statement of Financial Position is shown below.
January 1December 31
Accounts payable $ 32,000 $ 84,000
Accrued liabilities 14,000 11,000
7% bonds payable 95,000 77,000
Common stock ($10 par value) 300,000 300,000
Reserve for bond retirement 12,000 28,000
Retained earnings 155,000 206,000
Total liabilities and shareholders' equity $608,000 $706,000

Mica's debt/equity ratio is

A. 25.1%.
B. 25.6%.
C. 33.9%.
D. 32.2%.

Question 29 - ICMA 10.P2.041 - Ratios: Liquidity, Leverage, Coverage and Activity

The use of debt in the capital structure of a firm

A. decreases its financial leverage.


B. decreases its operating leverage.
C. increases its operating leverage.
D. increases its financial leverage.

Question 30 - CMA 692 2.27 - Ratios: Liquidity, Leverage, Coverage and Activity

If a company decided to change from the first-in, first-out (FIFO) inventory method to the last-in, first-out (LIFO) method
during a period of rising prices, its

A. Inventory turnover ratio would be reduced.


B. Cash flow would be decreased.
C. Current ratio would be reduced.
D. Debt-to-equity ratio would be decreased.

(c) HOCK international, page 10


Part 2 : 11/10/17 09:52:04

Question 31 - ICMA 13.P2.003 - Ratios: Liquidity, Leverage, Coverage and Activity

A company's cash ratio will decrease if the company

A. purchases materials on account.


B. receives cash by issuing a short-term note payable.
C. purchases commercial paper.
D. sells goods for cash at a selling price lower than cost.

Question 32 - ICMA 10.P2.033 - Ratios: Liquidity, Leverage, Coverage and Activity

The acid test ratio shows the ability of a company to pay its current liabilities without having to

A. reduce its cash balance.


B. collect its receivables.
C. borrow additional funds.
D. liquidate its inventory.

Question 33 - CMA 1294 2.22 - Ratios: Liquidity, Leverage, Coverage and Activity

The following inventory and sales data are available for the current year for Volpone Company. Volpone uses a
365-day year when computing ratios.
November 30, 2012 November 30, 2011
Net credit sales $6,205,000
Gross receivables 350,000 320,000
Inventory 960,000 780,000
Cost of goods sold 4,380,000

Volpone Company's average number of days to collect accounts receivable for the current year is

A. 19.71 days.
B. 18.87 days.
C. 21.17 days.
D. 19.43 days.

Question 34 - ICMA 13.P2.015 - Ratios: Liquidity, Leverage, Coverage and Activity

Since incorporating three years ago, Lawrence Inc. has estimated bad debts at a rate of 3% using the income
statement approach. During its fourth year in business, after recording the uncollectible accounts expense based on its
previous estimate, Lawrence determined that its estimate of bad debts should be increased to 4.5%. During this fourth
year, Lawrence recorded sales of $25,000,000 and had an ending accounts receivable balance of $2,000,000. This
change would decrease

A. the current year's income by $375,000 and increase the firm's degree of operating leverage.
B. the current year's income by $30,000 and decrease the firm's financial leverage.

(c) HOCK international, page 11


Part 2 : 11/10/17 09:52:04

C. both degree of operating leverage and times interest earned.


D. the current year's income by $1,125,000 and decrease the firm's degree of operating leverage.

Question 35 - CIA 1193 IV.46 - Ratios: Liquidity, Leverage, Coverage and Activity

The following account balances represent the end-of-year balance sheet of a company.
Accounts payable $ 67,000
Accounts receivable (net) 115,000
Accumulated depreciation - building 298,500
Accumulated depreciation - equipment 50,500
Cash 27,500
Common stock ($10 par value) 100,000
Deferred tax liabilities - noncurrent 37,500
Equipment 136,000
Income taxes payable 70,000
Inventory 257,000
Land and building 752,000
Long-term notes payable 123,000
Trading securities 64,000
Notes payable within 1 year 54,000
Other current liabilities 22,500
Paid-in capital in excess of par 150,000
Prepaid expenses 27,000
Retained earnings 403,500

The company's quick ratio is:

A. 0.97
B. 1.09
C. 0.82
D. 1.44

Question 36 - ICMA 10.P2.059 - Ratios: Liquidity, Leverage, Coverage and Activity

Lancaster Inc. had net accounts receivable of $168,000 and $147,000 at the beginning and end of the year,
respectively. The company’s net income for the year was $204,000 on $1,700,000 in total sales. Cash sales were 6%
of total sales. Lancaster's average accounts receivable turnover ratio for the year is

A. 10.15.
B. 10.79.
C. 9.51.
D. 10.87.

Question 37 - ICMA 10.P2.046 - Ratios: Liquidity, Leverage, Coverage and Activity

Which one of the following is the best indicator of long-term debt paying ability?

A. Current ratio.

(c) HOCK international, page 12


Part 2 : 11/10/17 09:52:04

B. Working capital turnover.


C. Debt-to-total assets ratio.
D. Asset turnover.

Question 38 - CMA 1280 4.2 - Ratios: Liquidity, Leverage, Coverage and Activity

Depoole Company is a manufacturer of industrial products and employs a calendar year for financial reporting
purposes. Assume that total quick assets exceeded total current liabilities both before and after the transaction
described. Further assume that Depoole has positive profits during the year and a credit balance throughout the year in
its retained earnings account.

The purchase of raw materials for $85,000 on open account would

A. Decrease the current ratio.


B. Decrease net working capital.
C. Increase net working capital.
D. Increase the current ratio.

Question 39 - CMA 679 4.13 - Ratios: Liquidity, Leverage, Coverage and Activity

Stock options are frequently provided to officers of companies. Stock options that are exercised improve

A. The ownership interest of existing stockholders.


B. Basic earnings per share.
C. The total asset turnover.
D. The debt-to-equity ratio.

Question 40 - ICMA 10.P2.052 - Ratios: Liquidity, Leverage, Coverage and Activity

Lowell Corporation has decided to include certain financial ratios in its year-end annual report to shareholders.
Selected information relating to its most recent fiscal year is provided below.
Cash $ 10,000
Accounts receivable (end of year) 20,000
Accounts receivable (beginning of year) 24,000
Inventory (end of year) 30,000
Inventory (beginning of year) 26,000
Notes payable (due in 90 days) 25,000
Bonds payable (due in 10 years) 35,000
Net credit sales for year 220,000
Cost of goods sold 140,000

Using a 365-day year, compute Lowell's accounts receivable turnover in days.

A. 36.5 days.
B. 26.1 days.
C. 33.2 days.
D. 39.8 days.

(c) HOCK international, page 13


Part 2 : 11/10/17 09:52:04

Question 41 - CIA 1195 IV.36 - Ratios: Liquidity, Leverage, Coverage and Activity

Which of the following financial ratios is used to assess the liquidity of a company?

A. Current Ratio.
B. Profit Margin on Sales.
C. Total Debt to Total Assets Ratio.
D. Days' Sales Outstanding.

Question 42 - ICMA 10.P2.058 - Ratios: Liquidity, Leverage, Coverage and Activity

Globetrade is a retailer that buys virtually all of its merchandise from manufacturers in a country experiencing
significant inflation. Globetrade is considering changing its method of inventory costing from first-in, first-out (FIFO) to
last-in, first-out (LIFO). What effect would the change from FIFO to LIFO have on Globetrade’s current ratio and
inventory turnover ratio?

A. Both the current ratio and the inventory turnover ratio would increase.
B. Both the current ratio and the inventory turnover ratio would decrease.
C. The current ratio would increase but the inventory turnover ratio would decrease.
D. The current ratio would decrease but the inventory turnover ratio would increase.

Question 43 - ICMA 13.P2.029 - Ratios: Liquidity, Leverage, Coverage and Activity

All other things being equal, which one of the following factors would result in an increase in cash reported on the
balance sheet from one period to the next?

A. Decrease in the accrued vacation liability.


B. Reduction of days sales outstanding of accounts receivable.
C. Increase in the speed with which accounts payable invoices are paid.
D. Increase in the level of inventory held.

Question 44 - CMA 692 1.10 - Ratios: Liquidity, Leverage, Coverage and Activity

Carlisle Company currently sells 400,000 bottles of perfume each year. Each bottle costs $0.84 to produce and sells
for $1.00. Fixed costs are $28,000 per year. The firm has annual interest expense of $6,000, preferred stock dividends
of $2,000 per year, and a 40% tax rate. Carlisle uses the following formulas to determine the company's leverage.

Operating leverage = [Q(S − VC)] ÷ [Q(S − VC) − FC]

Financial leverage = EBIT ÷ {EBIT − I − [P / (1 − T)]}

Total leverage = Q(S − VC) ÷ {Q(S − VC) − FC − I − [P / (1 − T)]}

Where:
Q=Quantity
FC=Fixed Cost
VC=Variable Cost
S=Selling Price

(c) HOCK international, page 14


Part 2 : 11/10/17 09:52:04

I=Interest Expense
P=Preferred Dividends
T=Tax Rate
EBIT=Earnings Before Interest and Taxes

If Carlisle Company did not have preferred stock, the degree of total leverage would

A. Decrease but not be proportional to the decrease in financial leverage.


B. Decrease in proportion to a decrease in financial leverage.
C. Increase in proportion to an increase in financial leverage.
D. Remain the same.

Question 45 - CMA 1280 4.7 - Ratios: Liquidity, Leverage, Coverage and Activity

Depoole Company is a manufacturer of industrial products and employs a calendar year for financial reporting
purposes. Assume that total quick assets exceeded total current liabilities both before and after the transaction
described. Further assume that Depoole has positive profits during the year and a credit balance throughout the year in
its retained earnings account.

The early liquidation of a long-term note with cash affects the

A. Current ratio but not the quick ratio.


B. Current ratio to a greater degree than the quick ratio.
C. Quick ratio to a greater degree than the current ratio.
D. Current and quick ratio to the same degree.

Question 46 - ICMA 10.P2.038 - Ratios: Liquidity, Leverage, Coverage and Activity

A summary of the Income Statement of Sahara Company is shown below.


Sales $15,000,000
Cost of sales 9,000,000
Operating expenses 3,000,000
Interest expense 800,000
Taxes 880,000
Net income $ 1,320,000

Based on the above information, Sahara's degree of financial leverage is

A. 1.36.
B. 0.96.
C. 1.61.
D. 2.27.

Question 47 - CMA 1291 1.5 - Ratios: Liquidity, Leverage, Coverage and Activity

The purchase of treasury stock with a firm's surplus cash

A. Increases a firm's equity.


B. Increases a firm's assets.

(c) HOCK international, page 15


Part 2 : 11/10/17 09:52:04

C. Increases a firm's financial leverage.


D. Increases a firm's interest-coverage ratio.

Question 48 - CMA 685 4.17 - Ratios: Liquidity, Leverage, Coverage and Activity

If the ratio of total liabilities to equity increases, a ratio that must also increase is

A. Return on equity.
B. Total liabilities to total assets.
C. The current ratio.
D. Times interest earned.

Question 49 - CMA 1280 4.6 - Ratios: Liquidity, Leverage, Coverage and Activity

Depoole Company is a manufacturer of industrial products and employs a calendar year for financial reporting
purposes. Assume that total quick assets exceeded total current liabilities both before and after the transaction
described. Further assume that Depoole has positive profits during the year and a credit balance throughout the year in
its retained earnings account.

The issuance of serial bonds in exchange for an office building, with the first installment of the bonds due late this year,

A. Affects all of the answers as indicated.


B. Decreases the quick ratio.
C. Decreases net working capital.
D. Decreases the current ratio.

Question 50 - ICMA 10.P2.031 - Ratios: Liquidity, Leverage, Coverage and Activity

When reviewing a credit application, the credit manager should be most concerned with the applicant's

A. profit margin and return on assets.


B. working capital and current ratio.
C. price-earnings ratio and current ratio.
D. working capital and return on equity.

Question 51 - ICMA 10.P2.028 - Ratios: Liquidity, Leverage, Coverage and Activity

Garstka Auto Parts must increase its acid test ratio above the current 0.9 level in order to comply with the terms of a
loan agreement. Which one of the following actions is most likely to produce the desired results?

A. Expediting collection of accounts receivable.


B. Selling auto parts on account.
C. Purchasing marketable securities for cash.
D. Making a payment to trade accounts payable.

(c) HOCK international, page 16


Part 2 : 11/10/17 09:52:04

Question 52 - ICMA 10.P2.063 - Ratios: Liquidity, Leverage, Coverage and Activity

The following information was obtained from a company’s financial statements.


Beginning of the
End of the Year
Year
Inventory $6,400 $7,600
Accounts receivable 2,140 3,060
Accounts payable 3,320 3,680

Total sales for the year were $85,900, of which $62,400 were credit sales. The cost of goods sold was $24,500. The
company's payable turnover was

A. 6.7 times.
B. 17.8 times.
C. 7.0 times.
D. 16.9 times.

Question 53 - CMA 1287 4.1 - Ratios: Liquidity, Leverage, Coverage and Activity

When a balance sheet amount is related to an income statement amount in computing a ratio,

A. The income statement amount should be converted to an average for the year.
B. Comparisons with industry ratios are not meaningful.
C. The balance sheet amount should be converted to an average for the year.
D. Both amounts should be converted to market value.

Question 54 - CMA 1280 4.4 - Ratios: Liquidity, Leverage, Coverage and Activity

Depoole Company is a manufacturer of industrial products and employs a calendar year for financial reporting
purposes. Assume that total quick assets exceeded total current liabilities both before and after the transaction
described. Further assume that Depoole has positive profits during the year and a credit balance throughout the year in
its retained earnings account.

Obsolete inventory of $125,000 was written off during the year. This transaction

A. Increased the quick ratio.


B. Decreased the quick ratio.
C. Decreased the current ratio.
D. Increased net working capital.

Question 55 - CMA 1291 1.9 - Ratios: Liquidity, Leverage, Coverage and Activity

Which one of the following factors would likely cause a firm to increase its use of debt financing as measured by the
debt-to-total-capitalization ratio?

A. An increase in the price-earnings ratio.


B. An increase in the degree of operating leverage.
C. An increase in the corporate income tax rate.

(c) HOCK international, page 17


Part 2 : 11/10/17 09:52:04

D. Increased economic uncertainty.

Question 56 - CMA 693 2.1 - Ratios: Liquidity, Leverage, Coverage and Activity

Lisa, Inc.
Statement of Financial Position
December 31, 20X4
(in thousands)
20X4 20X3
Assets
Current assets:
Cash $ 30 $ 25
Trading securities 20 15
Accounts receivable (net) 45 30
Inventories (at lower of cost of market) 60 50
Prepaid items 15 20
Total Current Assets $170 $140

Long-term assets:
Long-term investments:
Available-for-sale investments $ 25 $ 20
Property, plant & equipment:
Land (at cost) 75 75
Building (net) 80 90
Equipment (net) 95 100
Intangible assets:
Patents (net) 35 17
Goodwill (net) 20 13
Total Long-Term Assets $330 $315
Total Assets $500 $455

Liabilities and Equity


Current liabilities:
Notes payable $ 23 $ 12
Accounts payable 47 28
Accrued interest $ 15 $ 15
Total current liabilities $ 85 $ 55

Long-term liabilities:
Long-term Notes payable 10% due 12/31/20X6 $ 10 $ 10
Bonds payable 12% due 12/31/20X9 15 15
Total long-term debt $ 25 $ 25
Total liabilities $110 $ 80

Shareholders' Equity
Preferred stock - 5% cumulative, $100 par, nonparticipating
authorized, issued and outstanding, 1,000 shares $100 $100
Common stock - $10 par 20,000 shares authorized, 15,000
shares issued and outstanding 150 150
Additional paid-in capital - common 75 75
Retained earnings 65 50

(c) HOCK international, page 18


Part 2 : 11/10/17 09:52:04

Total Equity $390 $375


Total Liabilities & Equity $500 $455

Lisa Inc.'s acid test (quick) ratio at December 31, 20X4 was

A. 0.6
B. 1.1
C. 1.8
D. 2.0

Question 57 - ICMA 13.P2.010 - Ratios: Liquidity, Leverage, Coverage and Activity

A retail company has experienced rapid growth in sales during the current year. An analyst has calculated the following
ratios for this company.
Prior Year Current Year
Inventory Turnover 5.4 9.3
Receivables turnover 4.2 3.5
Fixed asset turnover 2.4 3.6
Quick ratio 1.5 1.2

Based on the above, the analyst may conclude that sales increased due to more

A. favorable credit policies.


B. stores open in current year.
C. control over inventory levels.
D. competitive pricing.

Question 58 - CIA 1196 IV.35 - Ratios: Liquidity, Leverage, Coverage and Activity

A company has a current ratio of 1.4, a quick, or acid test, ratio of 1.2, and the following partial summary balance sheet:
Cash $ 10 Current liabilities $
Accounts receivable ___ Long-term liabilities 40
Inventory ___ Shareholders' equity 30
Fixed Assets ___
Total assets $100 Total liabilities and equity$100

The company has a fixed assets balance of:

A. $58
B. $16
C. $64
D. $0

Question 59 - ICMA 10.P2.055 - Ratios: Liquidity, Leverage, Coverage and Activity

Peggy Monahan, controller, has gathered the following information regarding Lampasso Company.
Beginning of the year End of the year
Inventory $6,400 $7,600

(c) HOCK international, page 19


Part 2 : 11/10/17 09:52:04

Accounts receivable $2,140 $3,060


Accounts payable $3,320 $3,680

Total sales for the year were $85,900, of which $61,400 were credit sales. The cost of goods sold was $24,500.

Lampasso's inventory turnover ratio for the year was

A. 3.5 times.
B. 8.9 times.
C. 3.2 times.
D. 8.2 times.

Question 60 - CMA 695 2.1 - Ratios: Liquidity, Leverage, Coverage and Activity

CPZ Enterprises had the following account information.


Accounts receivable $200,000
Accounts payable 80,000
Bonds payable, due in ten years 10,000
Cash 100,000
Interest payable, due in three months 10,000
Inventory 400,000
Land 250,000
Notes payable, due in six months 50,000
Prepaid expenses 40,000

The company has an operating cycle of five months.

The current ratio for CPZ Enterprises is

A. 5.00
B. 5.29
C. 1.68
D. 2.14

Question 61 - ICMA 10.P2.039 - Ratios: Liquidity, Leverage, Coverage and Activity

A degree of operating leverage of 3 at 5,000 units means that a

A. 3% change in earnings before interest and taxes will cause a 3% change in sales.
B. 1% change in sales will cause a 3% change in earnings before interest and taxes.
C. 3% change in sales will cause a 3% change in earnings before interest and taxes.
D. 1% change in earnings before interest and taxes will cause a 3% change in sales.

Question 62 - ICMA 10.P2.061 - Ratios: Liquidity, Leverage, Coverage and Activity

The following financial information is given for Anjuli Corporation (in millions of dollars).
Prior Year Current Year
Sales $10 $11

(c) HOCK international, page 20


Part 2 : 11/10/17 09:52:04

Cost of goods sold 6 7


Current Assets:
Cash 2 3
Accounts receivable 3 4
Inventory 4 5

Between the prior year and the current year, did the days sales in inventory and days sales in receivables for Anjuli
increase or decrease? Assume a 365-day year.

A. Days sales in inventory increased; days sales in receivables decreased.


B. Days sales in inventory decreased; days sales in receivables decreased.
C. Days sales in inventory decreased; days sales in receivables increased.
D. Days sales in inventory increased; days sales in receivables increased.

Question 63 - CIA 1196 IV.34 - Ratios: Liquidity, Leverage, Coverage and Activity

A company has a current ratio of 1.4, a quick, or acid test, ratio of 1.2, and the following partial summary balance sheet:
Cash $ 10 Current liabilities $
Accounts receivable ___ Long-term liabilities 40
Inventory ___ Shareholders' equity 30
Fixed Assets ___
Total assets $100 Total liabilities and equity$100

The company has an accounts receivable balance of:

A. $12
B. $26
C. $36
D. $66

Question 64 - CMA 1289 P4 Q17 - Ratios: Liquidity, Leverage, Coverage and Activity

Excerpts from the statement of financial position for Landau Corporation as of September 30 of the current year are
presented as follows.
Cash $ 950,000
Accounts receivable (net) 1,675,000
Inventories 2,806,000
Total current assets $5,431,000
Accounts payable $1,004,000
Accrued liabilities 785,000
Total current liabilities $1,789,000

The board of directors of Landau Corporation met on October 4 of the current year and declared the regular quarterly
cash dividend amounting to $750,000 ($0.60 per share). The dividend is payable on October 25 of the current year to
all shareholders of record as of October 12 of the current year.

Assume that the only transactions to affect Landau Corporation during October of the current year are the dividend
transactions and that the closing entries have been made.

If the dividend declared by Landau Corporation had been a 10% stock dividend instead of a cash dividend, Landau's

(c) HOCK international, page 21


Part 2 : 11/10/17 09:52:04

total shareholders' equity would have been

A. Increased by the dividend declaration and unchanged by the dividend distribution.


B. Unchanged by the dividend declaration and increased by the dividend distribution.
C. Decreased by the dividend declaration and increased by the dividend distribution.
D. Unchanged by either the dividend declaration or the dividend distribution.

Question 65 - CMA 1285 4.23 - Ratios: Liquidity, Leverage, Coverage and Activity

Windham Company has current assets of $400,000 and current liabilities of $500,000. Windham Company's current
ratio would be increased by

A. The collection of $100,000 of accounts receivable.


B. The purchase of $100,000 of inventory on account.
C. Refinancing a $100,000 long-term loan with short-term debt.
D. The payment of $100,000 of accounts payable.

Question 66 - CMA 696 1.15 - Ratios: Liquidity, Leverage, Coverage and Activity

Spotech Co.'s budgeted sales and budgeted cost of sales for the coming year are $212,000,000 and $132,500,000,
respectively. Short-term interest rates are expected to average 5%. If Spotech could increase inventory turnover from
its current 8 times per year to 10 times per year, its expected cost savings in the current year would be

A. $331,250
B. $250,000
C. $82,812
D. $165,625

Question 67 - CMA 691 2.8 - Ratios: Liquidity, Leverage, Coverage and Activity

Selected data from Ostrander Corporation's financial statements for the years indicated are presented in thousands.

20X2 Operations
Net sales $4,175
Cost of goods sold 2,880
Interest expense 50
Income tax 120
Gain on disposal of a segment (net of tax) 210
Net income 385

December 31, 20X2


20X2 20X1
Cash $ 32 $ 28
Trading securities 169 172
Accounts receivable (net) 210 204
Merchandise inventory 440 420
Tangible fixed assets 480 440
Total assets 1,397 1,320

(c) HOCK international, page 22


Part 2 : 11/10/17 09:52:04

Current liabilities 370 368


Total liabilities 790 750
Common stock outstanding 226 210
Retained earnings 381 360

The total debt-to-equity ratio for Ostrander Corporation in 20X2 is

A. 1.85
B. 3.49
C. 1.30
D. 2.07

Question 68 - CMA 688 4.2 - Ratios: Liquidity, Leverage, Coverage and Activity

The data presented below shows actual figures for selected accounts of McKeon Company for the fiscal year ended
May 31, 20X0, and selected budget figures for the 20X1 fiscal year. McKeon's controller is in the process of reviewing
the 20X1 budget and calculating some key ratios based on the budget. McKeon Company monitors yield or return
ratios using the average financial position of the company. (Round all calculations to three decimal places if necessary.)
May 31, May 31,
20X1 20X0
Current assets $210,000 $180,000
Noncurrent assets 275,000 255,000
Current liabilities 78,000 85,000
Long-term debt 75,000 30,000
Common stock ($30 par value) 300,000 300,000
Retained earnings 32,000 20,000

20X1 Operations
Sales* $350,000
Cost of goods sold 160,000
Interest expense 3,000
Income taxes (40% rate) 48,000
Dividends declared and paid in 20X1 60,000
Administrative expense 67,000
*All sales are credit sales.

Composition of Current Assets


May 31, May 31,
20X1 20X0
Cash $ 20,000 $ 10,000
Accounts receivable 100,000 70,000
Inventory 70,000 80,000
Other 20,000 20,000
$210,000 $180,000

McKeon Company's debt to total asset ratio for 20X1 is

A. 0.315
B. 0.237
C. 0.264
D. 0.352

(c) HOCK international, page 23


Part 2 : 11/10/17 09:52:04

Question 69 - CMA 690 4.20 - Ratios: Liquidity, Leverage, Coverage and Activity

Assume the information below for Ramer Company, for Matson Company, and for their common industry represents a
recent year.
Industry
Ramer Matson Average
Current ratio 3.50 2.80 3.00
Accounts receivable turnover 5.00 8.10 6.00
Inventory turnover 6.20 8.00 6.10
Times interest earned 9.00 12.30 10.40
Debt-to-equity ratio 0.70 0.40 0.55
Return on investment 0.15 0.12 0.15
Dividend payout ratio 0.80 0.60 0.55
Earnings per share $3.00 $2.00 --

The attitudes of both Ramer and Matson concerning risk are best explained by the

A. Current ratio and earnings per share.


B. Dividend payout ratio and earnings per share.
C. Debt/equity ratio and times interest earned.
D. Current ratio, accounts receivable turnover, and inventory turnover.

Question 70 - ICMA 10.P2.056 - Ratios: Liquidity, Leverage, Coverage and Activity

Garland Corporation's Income Statement for the year just ended is shown below.
Net sales $900,000
Cost of goods sold:
Inventory - beginning $125,000
Purchases 540,000
Goods available for sale 665,000
Inventory - ending 138,000
Cost of goods sold 527,000
Gross profit 373,000
Operating expenses 175,000
Income from operations $198,000

Garland's average inventory turnover ratio is

A. 4.01.
B. 6.84.
C. 3.82.
D. 6.52.

Question 71 - CIA 596 IV.53 - Ratios: Liquidity, Leverage, Coverage and Activity

A growing company is assessing current working capital requirements. An average of 58 days is required to convert
raw materials into finished goods and to sell them. Then an average of 32 days is required to collect on receivables. If
the average time the company takes to pay for its raw materials is 15 days after they are received, then the total cash
conversion cycle for this company is:

(c) HOCK international, page 24


Part 2 : 11/10/17 09:52:04

A. 11 days.
B. 90 days.
C. 41 days.
D. 75 days.

Question 72 - CMA 693 2.4 - Ratios: Liquidity, Leverage, Coverage and Activity

Lisa, Inc.
Statement of Financial Position
December 31, 20X4
(in thousands)
20X4 20X3
Assets
Current assets:
Cash $ 30 $ 25
Trading securities 20 15
Accounts receivable (net) 45 30
Inventories (at lower of cost of market) 60 50
Prepaid items 15 20
Total Current Assets $170 $140

Long-term assets:
Long-term investments:
Available-for-sale investments $ 25 $ 20
Property, plant & equipment:
Land (at cost) 75 75
Building (net) 80 90
Equipment (net) 95 100
Intangible assets:
Patents (net) 35 17
Goodwill (net) 20 13
Total Long-Term Assets $330 $315
Total Assets $500 $455

Liabilities and Equity


Current liabilities:
Notes payable $ 23 $ 12
Accounts payable 47 28
Accrued interest $ 15 $ 15
Total current liabilities $ 85 $ 55

Long-term liabilities:
Long-term Notes payable 10% due 12/31/20X6 $ 10 $ 10
Bonds payable 12% due 12/31/20X9 15 15
Total long-term debt $ 25 $ 25
Total liabilities $110 $ 80

Shareholders' Equity
Preferred stock - 5% cumulative, $100 par, nonparticipating
authorized, issued and outstanding, 1,000 shares $100 $100
Common stock - $10 par 20,000 shares authorized, 15,000

(c) HOCK international, page 25


Part 2 : 11/10/17 09:52:04

shares issued and outstanding 150 150


Additional paid-in capital - common 75 75
Retained earnings 65 50
Total Equity $390 $375
Total Liabilities & Equity $500 $455

Assume sales and cost of goods sold for 20X4 were $300,000 and $220,000, respectively. Lisa Inc.'s inventory
turnover, using a 360-day year, was

A. 4.4 times.
B. 3.7 times.
C. 4.0 times.
D. 5.0 times.

Question 73 - ICMA 10.P2.030 - Ratios: Liquidity, Leverage, Coverage and Activity

Selected financial data for Boyd Corporation are shown below.


January 1 December 31
Cash $ 48,000 $ 62,000
Accounts receivable (net) 68,000 47,000
Trading securities 42,000 35,000
Inventory 125,000 138,000
Plant & equipment (net) 325,000 424,000
Accounts payable 32,000 84,000
Accrued liabilities 14,000 11,000
Deferred taxes 15,000 9,000
Long-term bonds payable 95,000 77,000

Boyd's net income for the year was $96,000. Boyd's current ratio at the end of the year is

A. 2.71.
B. 1.71.
C. 2.97.
D. 1.56.

Question 74 - ICMA 10.P2.032 - Ratios: Liquidity, Leverage, Coverage and Activity

Both the current ratio and the quick ratio for Spartan Corporation have been slowly decreasing. For the past two years,
the current ratio has been 2.3 to 1 and 2.0 to 1. During the same time period, the quick ratio has decreased from 1.2 to
1 to 1.0 to 1. The disparity between the current and quick ratios can be explained by which one of the following?

A. The cash balance is unusually low.


B. The accounts receivable balance has decreased.
C. The current portion of long-term debt has been steadily increasing.
D. The inventory balance is unusually high.

Question 75 - CIA 1193 IV.48 - Ratios: Liquidity, Leverage, Coverage and Activity

(c) HOCK international, page 26


Part 2 : 11/10/17 09:52:04

A company is considering the early retirement of its 10%, 10-year bonds payable. Before retiring the bonds, the
company's capital structure was
Current liabilities $125,000
Long-term liabilities: Notes payable (due in 5 years) 200,000
Bonds payable 300,000
Premium on bonds payable 25,000
Owner's equity: Common stock ($5 par value) 150,000
Paid-in capital in excess of par 50,000
Retained earnings 450,000

If the bonds can be retired at 103.5%, the

A. Asset turnover ratio will decrease.


B. Financial leverage will decrease.
C. Debt-equity ratio will increase.
D. Return on owner's equity will decrease.

Question 76 - CMA 1293 2.13 - Ratios: Liquidity, Leverage, Coverage and Activity

In computing inventory turnover, the base to use is the

A. Sales base because it more clearly represents operational activity.


B. Sales base because it is more likely to reflect a change in trend.
C. Cost of sales base because it eliminates any changes due solely to sales price changes.
D. Sales base because it provides turnover rates that are considerably higher.

Question 77 - CMA 688 4.3 - Ratios: Liquidity, Leverage, Coverage and Activity

The data presented below shows actual figures for selected accounts of McKeon Company for the fiscal year ended
May 31, 20X0, and selected budget figures for the 20X1 fiscal year. McKeon's controller is in the process of reviewing
the 20X1 budget and calculating some key ratios based on the budget. McKeon Company monitors yield or return
ratios using the average financial position of the company. (Round all calculations to three decimal places if necessary.)
May 31, May 31,
20X1 20X0
Current assets $210,000 $180,000
Noncurrent assets 275,000 255,000
Current liabilities 78,000 85,000
Long-term debt 75,000 30,000
Common stock ($30 par value) 300,000 300,000
Retained earnings 32,000 20,000

20X1 Operations
Sales* $350,000
Cost of goods sold 160,000
Interest expense 3,000
Income taxes (40% rate) 48,000
Dividends declared and paid in 20X1 60,000
Administrative expense 67,000
*All sales are credit sales.

(c) HOCK international, page 27


Part 2 : 11/10/17 09:52:04

Composition of Current Assets


May 31, May 31,
20X1 20X0
Cash $ 20,000 $ 10,000
Accounts receivable 100,000 70,000
Inventory 70,000 80,000
Other 20,000 20,000
$210,000 $180,000

The 20X1 accounts receivable turnover for McKeon Company is

A. 5.000
B. 1.882
C. 4.118
D. 3.500

Question 78 - CMA 688 4.11 - Ratios: Liquidity, Leverage, Coverage and Activity

A measure of its long-term debt-paying ability is a company's

A. Inventory turnover.
B. Times interest earned.
C. Return on assets.
D. Length of the operating cycle.

Question 79 - CMA 688 4.12 - Ratios: Liquidity, Leverage, Coverage and Activity

Using the data presented below, calculate the cost of sales for the Beta Corporation for the past year.
Current ratio 3.5
Acid test ratio 3.0
Current liabilities at year end $600,000
Beginning inventory $500,000
Inventory turnover 8.0

A. $2,400,000
B. $1,600,000
C. $3,200,000
D. $6,400,000

Question 80 - CMA 1280 4.1 - Ratios: Liquidity, Leverage, Coverage and Activity

Depoole Company is a manufacturer of industrial products and employs a calendar year for financial reporting
purposes. Assume that total quick assets exceeded total current liabilities both before and after the transaction
described. Further assume that Depoole has positive profits during the year and a credit balance throughout the year in
its retained earnings account.

Payment of a trade account payable of $64,500 would

(c) HOCK international, page 28


Part 2 : 11/10/17 09:52:04

A. Increase the current ratio but the quick ratio would not be affected.
B. Decrease both the current and quick ratios.
C. Increase the quick ratio but the current ratio would not be affected.
D. Increase both the current and quick ratios.

Question 81 - ICMA 10.P2.027 - Ratios: Liquidity, Leverage, Coverage and Activity

Shown below are selected data from Fortune Company's most recent financial statements.
Marketable securities $10,000
Accounts receivable 60,000
Inventory 25,000
Supplies 5,000
Accounts payable 40,000
Short-term debt payable 10,000
Accruals 5,000

What is Fortune's net working capital?

A. $35,000
B. $80,000
C. $50,000
D. $45,000

Question 82 - CMA 690 4.13 - Ratios: Liquidity, Leverage, Coverage and Activity

To determine the operating cycle for a wholesaler, which one of the following pairs of items is needed?

A. Asset turnover and return on sales.


B. Days' sales in accounts receivable and average merchandise inventory.
C. Cash turnover and net sales.
D. Accounts receivable turnover and inventory turnover.

Question 83 - HOCK MP1 E21 - Ratios: Liquidity, Leverage, Coverage and Activity

The annual sales revenue of an enterprise is $3,000,000. Half of the sales are on credit terms; half are cash sales.
Accounts receivable at the balance sheet date are $165,000. What is the average receivables collection period, to the
nearest day, using a 365 day year?

A. 9 days
B. 20 days
C. 40 days
D. 18 days

Question 84 - CMA 688 4.4 - Ratios: Liquidity, Leverage, Coverage and Activity

The data presented below shows actual figures for selected accounts of McKeon Company for the fiscal year ended

(c) HOCK international, page 29


Part 2 : 11/10/17 09:52:04

May 31, 20X0, and selected budget figures for the 20X1 fiscal year. McKeon's controller is in the process of reviewing
the 20X1 budget and calculating some key ratios based on the budget. McKeon Company monitors yield or return
ratios using the average financial position of the company. (Round all calculations to three decimal places if necessary.)
May 31, May 31,
20X1 20X0
Current assets $210,000 $180,000
Noncurrent assets 275,000 255,000
Current liabilities 78,000 85,000
Long-term debt 75,000 30,000
Common stock ($30 par value) 300,000 300,000
Retained earnings 32,000 20,000

20X1 Operations
Sales* $350,000
Cost of goods sold 160,000
Interest expense 3,000
Income taxes (40% rate) 48,000
Dividends declared and paid in 20X1 60,000
Administrative expense 67,000
*All sales are credit sales.

Composition of Current Assets


May 31, May 31,
20X1 20X0
Cash $ 20,000 $ 10,000
Accounts receivable 100,000 70,000
Inventory 70,000 80,000
Other 20,000 20,000
$210,000 $180,000

Using a 365-day year, McKeon's days of sales in inventory is

A. 160 days.
B. 183 days.
C. 171 days.
D. 78 days.

Question 85 - CMA 1280 4.3 - Ratios: Liquidity, Leverage, Coverage and Activity

Depoole Company is a manufacturer of industrial products and employs a calendar year for financial reporting
purposes. Assume that total quick assets exceeded total current liabilities both before and after the transaction
described. Further assume that Depoole has positive profits during the year and a credit balance throughout the year in
its retained earnings account.

The collection of a current accounts receivable of $29,000 would

A. Increase the current ratio.


B. Decrease the current ratio and the quick ratio.
C. Not affect the current or quick ratios.
D. Increase the quick ratio.

(c) HOCK international, page 30


Part 2 : 11/10/17 09:52:04

Question 86 - CMA 690 1.9 - Ratios: Liquidity, Leverage, Coverage and Activity

Sylvan Corporation has the following capital structure.


Debenture bonds: $10,000,000
Preferred equity: $1,000,000
Common equity: $39,000,000

The financial leverage of Sylvan Corporation would increase as a result of:

A. Issuing common stock and using the proceeds to retire preferred stock.
B. Financing its future investments with a higher percentage of bonds.
C. Maintaining the same dollar level of cash dividends as the prior year, even though earnings have increased by 7%.
D. Financing its future investments with a higher percentage of equity funds.

Question 87 - ICMA 10.P2.036 - Ratios: Liquidity, Leverage, Coverage and Activity

If a company has a current ratio of 2.1 and pays off a portion of its accounts payable with cash, the current ratio will

A. remain unchanged.
B. decrease.
C. increase.
D. move closer to the quick ratio.

Question 88 - CMA 695 1.1 - Ratios: Liquidity, Leverage, Coverage and Activity

A higher degree of operating leverage compared with the industry average implies that the firm

A. Has profits that are more sensitive to changes in sales volume.


B. Has higher variable costs.
C. Is more profitable.
D. Is less risky.

Question 89 - ICMA 10.P2.066 - Ratios: Liquidity, Leverage, Coverage and Activity

The assets of Moreland Corporation are presented below.


January 1 December 31
Cash $ 48,000 $ 62,000
Marketable securities 42,000 35,000
Accounts receivable 68,000 47,000
Inventory 125,000 138,000
Plant & equipment
(net of accumulated depreciation) 325,000 424,000

For the year just ended, Moreland had net income of $96,000 on $900,000 of sales. Moreland's total asset turnover
ratio is

A. 1.48.
B. 1.50.

(c) HOCK international, page 31


Part 2 : 11/10/17 09:52:04

C. 1.37.
D. 1.27.

Question 90 - ICMA 10.P2.034 - Ratios: Liquidity, Leverage, Coverage and Activity

All of the following are included when calculating the acid test ratio except

A. six-month treasury bills.


B. 60-day certificates of deposit.
C. prepaid insurance.
D. accounts receivable.

Question 91 - ICMA 10.P2.024 - Ratios: Liquidity, Leverage, Coverage and Activity

Shown below are beginning and ending balances for certain of Grimaldi Inc.'s accounts.
January 1 December 31
Cash $ 48,000 $ 62,000
Marketable securities 42,000 35,000
Accounts receivable 68,000 47,000
Inventory 125,000 138,000
Plant & equipment 325,000 424,000
Accounts payable 32,000 84,000
Accrued liabilities 14,000 11,000
7% bonds payable 95,000 77,000

Grimaldi's acid test ratio or quick ratio at the end of the year is

A. 1.02.
B. 0.83.
C. 1.52.
D. 1.15.

Question 92 - CIA 1190 IV.55 - Ratios: Liquidity, Leverage, Coverage and Activity

Assume that a company's total debt to total assets (debt-to-asset) ratio is currently 50%. It plans to purchase fixed
assets either by using borrowed funds for the purchase or by entering into an operating lease. The company's
debt-to-asset ratio as measured by the balance sheet will

A. Increase whether the assets are purchased or leased.


B. Increase if the assets are purchased, and remain unchanged if the assets are leased.
C. Remain unchanged whether the assets are purchased or leased.
D. Increase if the assets are purchased, and decrease if the assets are leased.

Question 93 - ICMA 10.P2.022 - Ratios: Liquidity, Leverage, Coverage and Activity

(c) HOCK international, page 32


Part 2 : 11/10/17 09:52:04

All of the following are affected when merchandise is purchased on credit except

A. current ratio.
B. total current assets.
C. total current liabilities.
D. net working capital.

Question 94 - HOCK MP1 E2 - Ratios: Liquidity, Leverage, Coverage and Activity

Ray Corporation has long-term debt of $1,200,000 and equity of $1,000,000. The board of directors has set a goal of
1:1 for the company's debt-equity ratio. Which of the following could the company employ to achieve this goal?

A. Issuing rights to purchase new common stock.


B. Issuing new bonds.
C. Paying a stock dividend to the existing shareholders.
D. Paying a dividend on its common stock.

Question 95 - CIA 594 IV.52 - Ratios: Liquidity, Leverage, Coverage and Activity

The degree of operating leverage (DOL) is

A. lower if the degree of total leverage is higher, other things held constant.
B. a measure of the change in earnings before interest and taxes (EBIT) resulting from a given change in sales.
C. higher if the degree of total leverage is lower, other things held constant.
D. a measure of the change in earnings available to common stockholders associated with a given change in operating
earnings.

Question 96 - CMA 695 2.3 - Ratios: Liquidity, Leverage, Coverage and Activity

CPZ Enterprises had the following account information.


Accounts receivable $200,000
Accounts payable 80,000
Bonds payable, due in ten years 10,000
Cash 100,000
Interest payable, due in three months 10,000
Inventory 400,000
Land 250,000
Notes payable, due in six months 50,000
Prepaid expenses 40,000

The company has an operating cycle of five months.

What will happen to the current and quick ratios if CPZ Enterprises uses cash to pay 50 percent of the accounts
payable?

A. Both ratios will increase.


B. The current ratio will decrease and the quick ratio will increase.
C. The current ratio will increase and the quick ratio will decrease.
D. Both ratios will decrease.

(c) HOCK international, page 33


Part 2 : 11/10/17 09:52:04

Question 97 - CMA 1293 2.16 - Ratios: Liquidity, Leverage, Coverage and Activity

The ratio that measures a firm's ability to generate earnings from its resources is

A. Asset turnover.
B. Sales to working capital.
C. Days' sales in inventory.
D. Days' sales in receivables.

Question 98 - CMA 693 2.3 - Ratios: Liquidity, Leverage, Coverage and Activity

Lisa, Inc.
Statement of Financial Position
December 31, 20X4
(in thousands)
20X4 20X3
Assets
Current assets:
Cash $ 30 $ 25
Trading securities 20 15
Accounts receivable (net) 45 30
Inventories (at lower of cost of market) 60 50
Prepaid items 15 20
Total Current Assets $170 $140

Long-term assets:
Long-term investments:
Available-for-sale investments $ 25 $ 20
Property, plant & equipment:
Land (at cost) 75 75
Building (net) 80 90
Equipment (net) 95 100
Intangible assets:
Patents (net) 35 17
Goodwill (net) 20 13
Total Long-Term Assets $330 $315
Total Assets $500 $455

Liabilities and Equity


Current liabilities:
Notes payable $ 23 $ 12
Accounts payable 47 28
Accrued interest $ 15 $ 15
Total current liabilities $ 85 $ 55

Long-term liabilities:
Long-term Notes payable 10% due 12/31/20X6 $ 10 $ 10
Bonds payable 12% due 12/31/20X9 15 15

(c) HOCK international, page 34


Part 2 : 11/10/17 09:52:04

Total long-term debt $ 25 $ 25


Total liabilities $110 $ 80

Shareholders' Equity
Preferred stock - 5% cumulative, $100 par, nonparticipating
authorized, issued and outstanding, 1,000 shares $100 $100
Common stock - $10 par 20,000 shares authorized, 15,000
shares issued and outstanding 150 150
Additional paid-in capital - common 75 75
Retained earnings 65 50
Total Equity $390 $375
Total Liabilities & Equity $500 $455

Assume net credit sales were $300,000 for 20X4. Lisa Inc.'s average collection period for 20X4, using a 360-day year,
was

A. 61 days.
B. 36 days.
C. 54 days.
D. 45 days.

Question 99 - ICMA 10.P2.023 - Ratios: Liquidity, Leverage, Coverage and Activity

Birch Products Inc. has the following current assets.


Cash $ 250,000
Marketable securities 100,000
Accounts receivable 800,000
Inventories 1,450,000
Total current assets $2,600,000

If Birch's current liabilities are $1,300,000, the firm's

A. current ratio will not change if a payment of $100,000 cash is used to pay $100,000 of accounts payable.
B. quick ratio will not change if a payment of $100,000 cash is used to purchase inventory.
C. quick ratio will decrease if a payment of $100,000 cash is used to purchase inventory.
D. current ratio will decrease if a payment of $100,000 cash is used to pay $100,000 of accounts payable.

Question 100 - CMA 1289 P4 Q14 - Ratios: Liquidity, Leverage, Coverage and Activity

Excerpts from the statement of financial position for Landau Corporation as of September 30 of the current year are
presented as follows.
Cash $ 950,000
Accounts receivable (net) 1,675,000
Inventories 2,806,000
Total current assets $5,431,000
Accounts payable $1,004,000
Accrued liabilities 785,000
Total current liabilities $1,789,000

The board of directors of Landau Corporation met on October 4 of the current year and declared the regular quarterly

(c) HOCK international, page 35


Part 2 : 11/10/17 09:52:04

cash dividend amounting to $750,000 ($0.60 per share). The dividend is payable on October 25 of the current year to
all shareholders of record as of October 12 of the current year.

Assume that the only transactions to affect Landau Corporation during October of the current year are the dividend
transactions and that the closing entries have been made.

Landau Corporation's current ratio was

A. Decreased by the dividend declaration and increased by the dividend payment.


B. Unchanged by either the dividend declaration or the dividend payment.
C. Decreased by the dividend declaration and unchanged by the dividend payment.
D. Increased by the dividend declaration and unchanged by the dividend payment.

Question 101 - ICMA 10.P2.029 - Ratios: Liquidity, Leverage, Coverage and Activity

The owner of a chain of grocery stores has bought a large supply of mangoes and paid for the fruit with cash. This
purchase will adversely impact which one of the following?

A. Price earnings ratio.


B. Working capital.
C. Quick or acid test ratio.
D. Current ratio.

Question 102 - CMA 690 4.14 - Ratios: Liquidity, Leverage, Coverage and Activity

Accounts receivable turnover will normally decrease as a result of

A. A change in credit policy to lengthen the period for cash discounts.


B. The write-off of an uncollectible account (assume the use of the allowance for doubtful accounts method).
C. An increase in cash sales in proportion to credit sales.
D. A significant sales volume decrease near the end of the accounting period.

Question 103 - CMA 1289 P4 Q13 - Ratios: Liquidity, Leverage, Coverage and Activity

Excerpts from the statement of financial position for Landau Corporation as of September 30 of the current year are
presented as follows.
Cash $ 950,000
Accounts receivable (net) 1,675,000
Inventories 2,806,000
Total current assets $5,431,000
Accounts payable $1,004,000
Accrued liabilities 785,000
Total current liabilities $1,789,000

The board of directors of Landau Corporation met on October 4 of the current year and declared the regular quarterly
cash dividend amounting to $750,000 ($0.60 per share). The dividend is payable on October 25 of the current year to
all shareholders of record as of October 12 of the current year.

Assume that the only transactions to affect Landau Corporation during October of the current year are the dividend

(c) HOCK international, page 36


Part 2 : 11/10/17 09:52:04

transactions and that the closing entries have been made.

Landau Corporation's working capital was

A. Decreased by the dividend declaration and increased by the dividend payment.


B. Unchanged by either the dividend declaration or the dividend payment.
C. Unchanged by the dividend declaration and decreased by the dividend payment.
D. Decreased by the dividend declaration and unchanged by the dividend payment.

Question 104 - ICMA 10.P2.043 - Ratios: Liquidity, Leverage, Coverage and Activity

Which one of the following statements concerning the effects of leverage on earnings before interest and taxes (EBIT)
and earnings per share (EPS) is correct?

A. Financial leverage affects both EPS and EBIT, while operating leverage only affects EBIT.
B. If Firm A has a higher degree of operating leverage than Firm B, and Firm A offsets this by using less financial
leverage, then both firms will have the same variability in EBIT.
C. A decrease in the financial leverage of a firm will increase the beta value of the firm.
D. For a firm using debt financing, a decrease in EBIT will result in a proportionally larger decrease in EPS.

Question 105 - ICMA 10.P2.048 - Ratios: Liquidity, Leverage, Coverage and Activity

The following information has been derived from the financial statements of Boutwell Company.
Current assets $640,000
Total assets 990,000
Long-term liabilities 130,000
Current ratio 3.2

The company's debt-to-equity ratio is

A. 0.50 to 1.
B. 0.13 to 1.
C. 0.33 to 1.
D. 0.37 to 1.

(c) HOCK international, page 37


Part 2 : 11/10/17 09:52:04

Question 1 - CIA 593 IV.40 - Ratios: Liquidity, Leverage, Coverage and Activity

A. Both ratios decreased in Year 2 compared to Year 1. See the correct answer for a complete explanation.

B. Both ratios decreased in Year 2 compared to Year 1. See the correct answer for a complete explanation.

C. Both ratios decreased in Year 2 compared to Year 1. See the correct answer for a complete explanation.

D.

In Year 1 the current ratio was 6.0 ($360 ÷ 60) and in Year 2 it was 4.3 ($430 ÷ $100). The current ratio has
therefore decreased.

In Year 1 the quick ratio was 2.67 ($160 ÷ $60) and in Year 2 it was 1.3 ($130 ÷ $100). The quick ratio has also
decreased.

Question 2 - ICMA 10.P2.037 - Ratios: Liquidity, Leverage, Coverage and Activity

A.

Leverage refers to how much debt a company has in its capital structure. Capital structure refers to the way a firm
chooses to finance its business, i.e., what proportion of its total capital (debt and equity) is in debt and what proportion
is in equity.

The most highly leveraged company is the company with the greatest proportion of debt in its capital structure. Cooper
is not the most highly leveraged of the four corporations.

B.

Leverage refers to how much debt a company has in its capital structure. Capital structure refers to the way a firm
chooses to finance its business, i.e., what proportion of its total capital (debt and equity) is in debt and what proportion
is in equity.

The most highly leveraged company is the company with the greatest proportion of debt in its capital structure. Pane is
not the most highly leveraged of the four corporations.

C.

Leverage refers to how much debt a company has in its capital structure. Capital structure refers to the way a
firm chooses to finance its business, i.e., what proportion of its total capital (debt and equity) is in debt and
what proportion is in equity.

The most highly leveraged company is the company with the greatest proportion of debt in its capital
structure. Looking at the percentages of long-term debt for the four companies, we see that Sterling has 40%
of its capital structure in debt, Cooper has 35%, Warwick has 30%, and Pane also has 30%. Based on that,
Sterling is the most highly leveraged corporation.

Leverage can also include short-term debt, because short-term debt is also used to fund assets. Whether
short-term debt is included or not really depends on the facts given. Here, the percentages of short-term debt
have been included in the calculation of 100% of the firms' capital structures, so it would be best to include
short-term debt in the calculation of debt in the capital structure. When we sum the short-term and long-term
debt for each company, we see that Sterling has 50% debt, Cooper has 45% debt, Warwick has 45% debt, and
Pane has 40% debt. Using both short-term and long-term debt, Sterling is again the most highly leveraged of
the four corporations.

(c) HOCK international, page 38


Part 2 : 11/10/17 09:52:04

D.

Leverage refers to how much debt a company has in its capital structure. Capital structure refers to the way a firm
chooses to finance its business, i.e., what proportion of its total capital (debt and equity) is in debt and what proportion
is in equity.

The most highly leveraged company is the company with the greatest proportion of debt in its capital structure.
Warwick is not the most highly leveraged of the four corporations.

Question 3 - ICMA 10.P2.035 - Ratios: Liquidity, Leverage, Coverage and Activity

A. The quick or acid test ratio is Cash + Marketable Securities + Accounts Receivable divided by Current Liabilities.
This answer results from including Inventory in the numerator.

B. The quick or acid test ratio is Cash + Marketable Securities + Accounts Receivable divided by Current Liabilities.
This answer results from including Prepaid Expenses and Inventory in the numerator.

C. The quick or acid test ratio is Cash + Marketable Securities + Accounts Receivable divided by Current
Liabilities. The numerator of the ratio is Cash ($10,000) + Accounts Receivable ($20,000) + Available-for-sale
Securities ($12,000 at fair value), for a total of $42,000. The denominator is Accounts Payable ($15,000) + Notes
payable due in 90 days ($25,000), for a total of $40,000. $42,000 ÷ $40,000 = 1.05.

D. The quick or acid test ratio is Cash + Marketable Securities + Accounts Receivable divided by Current Liabilities.
This answer results from including Available-for-sale Securities in the numerator at cost instead of at fair value, and
also including Prepaid Expenses and Inventory in the numerator.

Question 4 - CMA 692 1.9 - Ratios: Liquidity, Leverage, Coverage and Activity

A.

This is not the correct answer. Please see the correct answer for an explanation.

We have been unable to determine how to calculate this incorrect answer choice. If you have calculated it, please let
us know how you did it so we can create a full explanation of why this answer choice is incorrect. Please send us an
email at support@hockinternational.com. Include the full Question ID number and the actual incorrect answer choice --
not its letter, because that can change with every study session created. The Question ID number appears in the upper
right corner of the ExamSuccess screen. Thank you in advance for helping us to make your HOCK study materials
better.

B. Degree of financial leverage can be calculated as % Change in Net Income ÷ % Change in Operating Profit
(EBIT). When only one year of financial results is available, it can also be calculated as EBIT ÷ (EBIT −
Interest). When there is preferred stock, the second formula is modified as given in the problem: EBIT ÷ (EBIT
− I − [P / (1 − T)])

Earnings before interest and taxes equal $36,000 [$400,000 sales − ($0.84 x 400,000 units) VC − $28,000 FC].

Using the formula given in the problem, the calculation is as follows:

DFL = $36,000 ÷ [$36,000 − $6,000 − ($2,000 ÷ 0.6)]

= $36,000 ÷ $26,667

(c) HOCK international, page 39


Part 2 : 11/10/17 09:52:04

= 1.35

C. This is the degree of total leverage.

D. This is the degree of operating leverage, not financial leverage.

Question 5 - ICMA 10.P2.040 - Ratios: Liquidity, Leverage, Coverage and Activity

A. The Degree of Financial Leverage is the factor by which net income changes when related to a change in
earnings before interest and tax, since interest on debt is a fixed expense. If a company has a high degree of
financial leverage, it has a high amount of debt in its capital structure and thus would also have a high ratio of
debt to equity.

B.

A zero-coupon bond is a bond bought by an investor at a deep discount to its face value that repays its full face value
at maturity. The investor does not receive regular interest payments. Instead, the difference between the face value of
the bond and the investor's purchase price is interest to the investor, and it is received at the maturity of the bond.
Examples of zero-coupon bonds include U.S. Treasury bills, U.S. savings bonds, and any type of coupon bond that has
been stripped of its coupons, where the seller has retained the coupons and thus has the right to receive the regular
interest payments made by the issuer.

There is no connection between a high degree of financial leverage and zero coupon bonds.

C. A company with a high degree of financial leverage may have a high current ratio or a low current ratio. There is no
necessary connection between a high degree of financial leverage and a low current ratio.

D. The fixed-charge coverage ratio is earnings before fixed charges and taxes divided by fixed charges. A company
with a high degree of financial leveerage may have a high fixed-charge coverage ratio or a low fixed-charge coverage
ratio. There is no necessary connection between a high degree of financial leverage and a high fixed-charge coverage
ratio.

Question 6 - CMA 695 2.2 - Ratios: Liquidity, Leverage, Coverage and Activity

A. The quick ratio is calculated as quick assets (current assets excluding inventory and prepaid expenses)
divided by current liabilities. Quick assets include accounts receivable ($200,000) and cash ($100,000). Current
liabilities include accounts payable ($80,000), interest payable ($10,000) and notes payable ($50,000). Note that
even though the notes payable are due after the operating cycle is over, the distinction for a current asset or
liability is that it will be converted or settled within 12 months or the operating cycle, whichever is longer.
Thus the 12-month limit governs what is considered a current liability, not the 5-month length of the firm's
operating cycle. The total current assets are $300,000 and current liabilities are $140,000. This gives us a quick
ratio of 2.14.

B. The quick ratio is calculated as quick assets (current assets excluding inventory and prepaid expenses) divided by
current liabilities. This answer includes inventory and prepaid expenses as quick assets and includes bonds payable as
a current liability.

C. The quick ratio is calculated as quick assets (current assets excluding inventory and prepaid expenses) divided by
current liabilities. This answer includes bonds payable as a current liability.

D. The quick ratio is calculated as quick assets (current assets excluding inventory and prepaid expenses) divided by
current liabilities. This answer does not include the interest payable as a current liability.

(c) HOCK international, page 40


Part 2 : 11/10/17 09:52:04

Question 7 - CMA 693 2.2 - Ratios: Liquidity, Leverage, Coverage and Activity

A. This answer incorrectly uses cost of goods sold (instead of credit sales) in the numerator.

B. Accounts receivable turnover is calculated as net credit sales divided by average accounts receivable.
Average accounts receivable was $37,500 (the average of $45,000 and $30,000), and given information that
credit sales were $300,000, we get a receivables turnover of 8.

C. This answer includes only the year end receivables in the denominator (instead of the average receivables) and
uses cost of goods sold (instead of credit sales) in the numerator.

D. This answer incorrectly uses ending receivables (instead of average receivables) in the numerator.

Question 8 - ICMA 10.P2.060 - Ratios: Liquidity, Leverage, Coverage and Activity

A.

This is not the correct answer. Please see the correct answer for an explanation.

We have been unable to determine how to calculate this incorrect answer choice. If you have calculated it, please let
us know how you did it so we can create a full explanation of why this answer choice is incorrect. Please send us an
email at support@hockinternational.com. Include the full Question ID number and the actual incorrect answer choice --
not its letter, because that can change with every study session created. The Question ID number appears in the upper
right corner of the ExamSuccess screen. Thank you in advance for helping us to make your HOCK study materials
better.

B.

This is not the correct answer. Please see the correct answer for an explanation.

We have been unable to determine how to calculate this incorrect answer choice. If you have calculated it, please let
us know how you did it so we can create a full explanation of why this answer choice is incorrect. Please send us an
email at support@hockinternational.com. Include the full Question ID number and the actual incorrect answer choice --
not its letter, because that can change with every study session created. The Question ID number appears in the upper
right corner of the ExamSuccess screen. Thank you in advance for helping us to make your HOCK study materials
better.

C.

This answer results from using year-end accounts receivable instead of the average balance in accounts receivable
during the year.

The number of days' sales in accounts receivable can be calculated in two different ways, both of which are two-step
processes.

(1) Calculate the accounts receivable turnover ratio, then divide the number of days in the year by the accounts
receivable turnover ratio. The accounts receivable turnover ratio is net credit sales divided by average accounts
receivable.

(2) Divide the annual credit sales by the number of days in the year to find the amount of credit sales made per day.
Then divide average accounts receivable by the amount of credit sales made per day to find the number of days' sales
in receivables.

(c) HOCK international, page 41


Part 2 : 11/10/17 09:52:04

D.

The number of days' sales in accounts receivable can be calculated in two different ways, both of which are
two-step processes.

(1) Calculate the accounts receivable turnover ratio, then divide the number of days in the year by the
accounts receivable turnover ratio. The accounts receivable turnover ratio is net credit sales divided by
average accounts receivable. Average accounts receivable is the average of the beginning and ending
receivable balances, or ($68,000 + $47,000) ÷ 2, which equals $57,500. Since we are not told that Cornwall
made any cash sales, we must assume that all sales were on credit. The accounts receivable turnover ratio is
$900,000 ÷ $57,500, which equals 15.65 times. 360 ÷ 15.65 = 23 days.

(2) Divide the annual credit sales by the number of days in the year to find the amount of credit sales made per
day. Then divide average accounts receivable by the amount of credit sales made per day to find the number
of days' sales in receivables. $900,000 ÷ 360 = $2,500. $57,500 ÷ $2,500 = 23 days.

You can use either of these formulas to find the number of days' sales in receivables. Whichever one makes
the most sense to you is the one you should use, because you will have less trouble learning it.

Question 9 - CMA 1294 2.23 - Ratios: Liquidity, Leverage, Coverage and Activity

A. The average number of days to sell inventory is calculated as 365 divided by the inventory turnover ratio.
Inventory turnover is calculated as the cost of goods sold divided by average inventory. Cost of goods sold
was $4,380,000 and average inventory was $870,000 (the average of the beginning and ending balances). This
gives an inventory turnover of 5.03. Dividing 365 by 5.03 gives us 72.5 days as the average number of days to
sell inventory.

B. The average number of days to sell inventory is calculated as 365 divided by the inventory turnover ratio. Inventory
turnover is calculated as the cost of goods sold divided by average inventory (the average of the beginning and ending
balances). This answer uses only ending inventory in the calculation of inventory turnover.

C. The average number of days to sell inventory is calculated as 365 divided by the inventory turnover ratio. Inventory
turnover is calculated as the cost of goods sold divided by average inventory. This answer uses sales instead of the
cost of goods sold to calculate the inventory turnover.

D. The average number of days to sell inventory is calculated as 365 divided by the inventory turnover ratio. Inventory
turnover is calculated as the cost of goods sold divided by average inventory. This answer uses only beginning
inventory in the calculation of inventory turnover.

Question 10 - ICMA 10.P2.053 - Ratios: Liquidity, Leverage, Coverage and Activity

A. The accounts receivable turnover ratio is net credit sales divided by average accounts receivable. Average
accounts receivable is the average of the beginning and ending balances, which is $360,000. Net credit sales
is $3,600,000, so the accounts receivable turnover ratio is $3,600,000 ÷ $360,000, which equals 10. Accounts
receivable "turns over" 10 times during the year.

B.

This is net credit sales for the year divided by year end accounts receivable. The accounts receivable turnover ratio is
net credit sales divided by average accounts receivable.

(c) HOCK international, page 42


Part 2 : 11/10/17 09:52:04

Whenever we relate an income statement amount such as net credit sales to a balance sheet amount such as
accounts receivable, we need to use the average balance of the balance sheet amount, not the beginning or ending
balance.

C. This is average accounts receivable divided by net credit sales for the year. The accounts receivable turnover ratio
is net credit sales divided by average accounts receivable.

D.

This is net credit sales for the year divided by the beginning of the year accounts receivable balance. The accounts
receivable turnover ratio is net credit sales divided by average accounts receivable.

Whenever we relate an income statement amount such as net credit sales to a balance sheet amount such as
accounts receivable, we need to use the average balance of the balance sheet amount, not the beginning or ending
balance.

Question 11 - ICMA 10.P2.026 - Ratios: Liquidity, Leverage, Coverage and Activity

A. Increasing the allowance for uncollectible accounts is done by recording a debit to an expense account
(bad debt expense) and a credit to the allowance for uncollectible accounts, a valuation account on the
balance sheet that reduces accounts receivable. Thus, accounts receivable and current assets will decrease
as a result. The current ratio is current assets divided by current liabilities. A decrease in current assets (the
numerator) while current liabilities (the denominator) remain the same leads to a reduction in the current ratio.

B. Increasing the allowance for uncollectible accounts is done by recording a debit to an expense account (bad debt
expense) and a credit to the allowance for uncollectible accounts, a valuation account on the balance sheet that
reduces accounts receivable. Increasing expenses will cause a decrease in net income and thus in equity. It will also
cause a decrease in accounts receivable and total assets. Total debt will not change. When debt (the numerator)
remains the same while assets (the denominator) decrease, the ratio increases rather than decreases.

C. Increasing the allowance for uncollectible accounts is done by recording a debit to an expense account (bad debt
expense) and a credit to the allowance for uncollectible accounts, a valuation account on the balance sheet that
reduces accounts receivable. The acid test ratio (also called the quick ratio) is Cash + Marketable Securities +
Accounts Receivable divided by Current Liabilities. Reducing accounts receivable will reduce the numerator of the ratio
while the denominator remains the same. The result will be a decrease in the acid test ratio, not an increase.

D. Increasing the allowance for uncollectible accounts is done by recording a debit to an expense account (bad debt
expense) and a credit to the allowance for uncollectible accounts, a valuation account on the balance sheet that
reduces accounts receivable. Working capital is current assets minus current liabilities. Reducing accounts receivable
will reduce current assets, while current liabilities will not change. This will result in a decrease in working capital, not
an increase.

Question 12 - ICMA 10.P2.042 - Ratios: Liquidity, Leverage, Coverage and Activity

A.

There are two ways to calculate Degree of Financial Leverage, and this question can be answered using either
one of them.

(1) DFL = % Change in Net Income ÷ % Change in EBIT. To answer this question using this formula, we plug
what we know into the formula and solve for the unknown, which is % Change in Net Income, as follows:

(c) HOCK international, page 43


Part 2 : 11/10/17 09:52:04

1.5 = X / 0.05

Solving for X, we multiply both sides of the equation by 0.05:

1.5 × 0.05 = X

X = 0.075 or 7.50%

(2) DFL = EBIT ÷ EBT.

The best way to solve this is to make up some numbers: EBIT is $135,000; interest expense is $45,000; and
EBT is $135,000 − $45,000, which equals $90,000. Degree of Financial Leverage is $135,000 ÷ $90,000, which is
the 1.5 that it needs to be.

If EBIT increases by 5%, it becomes $141,750 ($135,000 × 1.05). Interest expense is a fixed cost, so it remains
$45,000. EBT is now $141,750 − $45,000, which equals $96,750.

To answer the question, we next need to find the percentage of increase in EBT. EBT now is $96,750, and
formerly it was $90,000, an increase of $6,750. $6,750 ÷ $90,000 = 0.075, so EBT has increased by 7.50%.

B. When a firm is using financial leverage successfully, the percentage of increase in earnings before taxes will be
greater than the percentage of increase in earnings before interest and taxes, not the same as the percentage of
increase in earnings before interest and taxes.

C. When a firm is using financial leverage successfully, the percentage of increase in earnings before taxes will be
greater than the percentage of increase in earnings before interest and taxes, not lower than the percentage of
increase in earnings before interest and taxes.

D. When a firm is using financial leverage successfully, the percentage of increase in earnings before taxes will be
greater than the percentage of increase in earnings before interest and taxes, not lower than the percentage of
increase in earnings before interest and taxes.

Question 13 - ICMA 10.P2.049 - Ratios: Liquidity, Leverage, Coverage and Activity

A.

This is not the correct answer. Please see the correct answer for an explanation.

We have been unable to determine how to calculate this incorrect answer choice. If you have calculated it, please let
us know how you did it so we can create a full explanation of why this answer choice is incorrect. Please send us an
email at support@hockinternational.com. Include the full Question ID number and the actual incorrect answer choice --
not its letter, because that can change with every study session created. The Question ID number appears in the upper
right corner of the ExamSuccess screen. Thank you in advance for helping us to make your HOCK study materials
better.

B. This answer results from reducing Earnings Before Interest and Taxes (EBIT) by income tax due on it based on the
tax rate. in calculating the times interest earned (interest coverage) ratio, EBIT should not be reduced by income tax
due on it.

C.

This is not the correct answer. Please see the correct answer for an explanation.

We have been unable to determine how to calculate this incorrect answer choice. If you have calculated it, please let
us know how you did it so we can create a full explanation of why this answer choice is incorrect. Please send us an

(c) HOCK international, page 44


Part 2 : 11/10/17 09:52:04

email at support@hockinternational.com. Include the full Question ID number and the actual incorrect answer choice --
not its letter, because that can change with every study session created. The Question ID number appears in the upper
right corner of the ExamSuccess screen. Thank you in advance for helping us to make your HOCK study materials
better.

D. The times interest earned ratio (interest coverage ratio) is EBIT ÷ Interest Expense. Since EBIT and Interest
Expense are the same, the company’s times interest earned ratio must be 1.0.

Question 14 - ICMA 10.P2.054 - Ratios: Liquidity, Leverage, Coverage and Activity

A. The receivables turnover ratio is net credit sales ÷ average accounts receivable. If sales decrease while average
inventory remains the same, the ratio will decrease, not increase.

B.

The inventory turnover ratio is annual cost of sales ÷ average inventory. If cost of sales decreases while average
inventory remains the same, the ratio will decrease, not increase.

The receivables turnover ratio is net credit sales ÷ average accounts receivable. If sales decrease while average
inventory remains the same, the ratio will decrease, not increase.

C.

The inventory turnover ratio is annual cost of sales ÷ average inventory. If cost of sales decreases while
average inventory remains the same, the ratio will decrease.

The receivables turnover ratio is net credit sales ÷ average accounts receivable. If sales decrease while
average accounts receivable increases, the ratio will decrease.

D. The inventory turnover ratio is annual cost of sales ÷ average inventory. If cost of sales decreases while average
inventory remains the same, the ratio will decrease, not increase.

Question 15 - ICMA 10.P2.062 - Ratios: Liquidity, Leverage, Coverage and Activity

A.

The fixed asset turnover ratio is sales divided by average net fixed assets. ("Net" means net of accumulated
depreciation.) In this problem, we do not have enough information to calculate average net fixed assets, so we must
use the year-end information given.

This is sales divided by total assets, not net fixed assets.

B.

The fixed asset turnover ratio is sales divided by average net fixed assets. ("Net" means net of accumulated
depreciation.) In this problem, we do not have enough information to calculate average net fixed assets, so we must
use the year-end information given.

This is net fixed assets divided by sales.

C.

(c) HOCK international, page 45


Part 2 : 11/10/17 09:52:04

The fixed asset turnover ratio is sales divided by average net fixed assets. ("Net" means net of accumulated
depreciation.) In this problem, we do not have enough information to calculate average net fixed assets, so we must
use the year-end information given.

This is total assets divided by sales.

D.

The fixed asset turnover ratio is sales divided by average net fixed assets. ("Net" means net of accumulated
depreciation.) In this problem, we do not have enough information to calculate average net fixed assets, so we
must use the year-end information given.

Sales of $3,000,000 divided by net fixed assets of $1,300,000 equals 2.31. The company is generating 2.3 times
its net fixed assets in sales.

Question 16 - CMA 690 1.16 - Ratios: Liquidity, Leverage, Coverage and Activity

A. Degree of financial leverage is calculated as the change in net income (after interest and taxes) divided by
the change in earnings before interest and taxes. This is 42% ÷ 17%, or 2.47.

B.

This is not the correct answer. Please see the correct answer for an explanation.

We have been unable to determine how to calculate this incorrect answer choice. If you have calculated it, please let
us know how you did it so we can create a full explanation of why this answer choice is incorrect. Please send us an
email at support@hockinternational.com. Include the full Question ID number and the actual incorrect answer choice --
not its letter, because that can change with every study session created. The Question ID number appears in the upper
right corner of the ExamSuccess screen. Thank you in advance for helping us to make your HOCK study materials
better.

C.

This is not the correct answer. Please see the correct answer for an explanation.

We have been unable to determine how to calculate this incorrect answer choice. If you have calculated it, please let
us know how you did it so we can create a full explanation of why this answer choice is incorrect. Please send us an
email at support@hockinternational.com. Include the full Question ID number and the actual incorrect answer choice --
not its letter, because that can change with every study session created. The Question ID number appears in the upper
right corner of the ExamSuccess screen. Thank you in advance for helping us to make your HOCK study materials
better.

D.

This is not the correct answer. Please see the correct answer for an explanation.

We have been unable to determine how to calculate this incorrect answer choice. If you have calculated it, please let
us know how you did it so we can create a full explanation of why this answer choice is incorrect. Please send us an
email at support@hockinternational.com. Include the full Question ID number and the actual incorrect answer choice --
not its letter, because that can change with every study session created. The Question ID number appears in the upper
right corner of the ExamSuccess screen. Thank you in advance for helping us to make your HOCK study materials
better.

(c) HOCK international, page 46


Part 2 : 11/10/17 09:52:04

Question 17 - CIA 590 IV.47 - Ratios: Liquidity, Leverage, Coverage and Activity

A. This answer adds the value of inventory to the current assets figure instead of subtracting it.

B. This answer incorrectly includes the value of the inventory in the numerator of the calculation. Inventory is included
in the current ratio, but not the acid test ratio.

C. This answer multiplies the numerator by 2 instead of dividing it by 2.

D.

The acid test ratio is calculated as follows: (Cash + Marketable Securities + Receivables) / Current Liabilities.
Inventory is a current asset, but it is not included in the acid test ratio calculation. This means that based on
the information given here, the numerator is only $3,000 ($5,000 current assets − $2,000 inventory). Since the
ratio is 2, the current liabilities must be $1,500.

Note that the full information needed to calculate the numerator of the acid test ratio is not given in this
problem. We must assume that there are no prepaids or other current assets such as short-term deferred tax
assets in current assets in order to calculate the numerator this way. Prepaids and other current assets are
not included in the numerator of the acid test, or quick, ratio. However, since total current assets and
inventory are the only information given for calculating the numerator, we must use what we have. But if the
individual components of current assets are given, the proper way to calculate the numerator is Cash +
Marketable Securities + Receivables, excluding any and all other current assets.

Question 18 - CMA 692 1.8 - Ratios: Liquidity, Leverage, Coverage and Activity

A. This is the degree of financial leverage.

B. Operating leverage is the percentage change in operating income resulting from a percentage change in
sales. When only one year of data is available, it is calculated as the contribution margin ÷ operating income
(EBIT), and that is the formula given here. Based on a contribution margin of $0.16 per unit ($1.00 − $0.84
variable cost), the degree of operating leverage is (400,000 x $0.16) ÷ [(400,000 x $0.16) − 28,000] = 1.78.

C.

This is not the correct answer. Please see the correct answer for an explanation.

We have been unable to determine how to calculate this incorrect answer choice. If you have calculated it, please let
us know how you did it so we can create a full explanation of why this answer choice is incorrect. Please send us an
email at support@hockinternational.com. Include the full Question ID number and the actual incorrect answer choice --
not its letter, because that can change with every study session created. The Question ID number appears in the upper
right corner of the ExamSuccess screen. Thank you in advance for helping us to make your HOCK study materials
better.

D. This is the degree of total leverage.

Question 19 - ICMA 10.P2.057 - Ratios: Liquidity, Leverage, Coverage and Activity

A. The inventory turnover ratio is Cost of Goods Sold ÷ Average Inventory. This is Cost of Goods Sold ÷ Ending
Inventory.

B. The inventory turnover ratio is Cost of Goods Sold ÷ Average Inventory. Average inventory is the average

(c) HOCK international, page 47


Part 2 : 11/10/17 09:52:04

of the beginning and ending inventories, which is ($30,000 + $26,000) ÷ 2, or $28,000. So the inventory
turnover ratio is $140,000 ÷ $28,000, which equals 5.0 times.

C. The inventory turnover ratio is Cost of Goods Sold ÷ Average Inventory. This is Cost of Goods Sold ÷ Beginning
Inventory.

D. The inventory turnover ratio is Cost of Goods Sold ÷ Average Inventory. This is Sales ÷ Average Inventory.

Question 20 - CMA 1295 2.13 - Ratios: Liquidity, Leverage, Coverage and Activity

A. Liquidity ratios have to do with the ability of the company to pay its short-term liabilities as the become due. Activity
ratios measure the ability of the company to manage its resources efficiently, specifically the current assets of accounts
receivable and inventory, and to manage its accounts payable effectively. The accounts receivable turnover ratio is a
measure of activity.

B. Liquidity ratios have to do with the ability of the company to pay its short-term liabilities as the become due. Activity
ratios measure the ability of the company to manage its resources efficiently, specifically the current assets of accounts
receivable and inventory, and to manage its accounts payable effectively. The average collection period is a measure
of activity.

C. Liquidity ratios have to do with the ability of the company to pay its short-term liabilities as the become due. Activity
ratios measure the ability of the company to manage its resources efficiently, specifically the current assets of accounts
receivable and inventory, and to manage its accounts payable effectively. The merchandise inventory turnover is a
measure of activity.

D. Liquidity ratios have to do with the ability of the company to pay its short-term liabilities as the become
due. Activity ratios measure the ability of the company to manage its resources efficiently, specifically the
current assets of accounts receivable and inventory, and to manage its accounts payable effectively. Times
interest earned measures neither the ability to pay current liabilities nor the company's management of its
resources and its accounts payable.

Question 21 - CMA 1293 2.17 - Ratios: Liquidity, Leverage, Coverage and Activity

A. Writing off a receivable will decrease both the receivables account and the related allowance for bad debts.
Therefore, net receivables will remain unchanged and the current ratio will remain unchanged.

B. A distribution of a stock dividend does not affect current assets or current liabilities because it is distributed in stock,
not cash. Therefore, the current ratio would remain unchanged.

C. When a company purchases inventory on account, both the current assets and the current liabilities of the company
will increase by the cost of the inventory. Since the company currently has a current ratio of 2, an equal increase in the
numerator and denominator will actually decrease the current ratio of the company because the percentage increase
in current assets will be less than the percentage increase in current liabilities.

D. When the company sells inventory at a profit, current assets will increase because the receivable that is
created is larger than the inventory that was sold (the difference is the profit on the transaction). This increase
in current assets will increase the current ratio.

Question 22 - CIA 595 IV.51 - Ratios: Liquidity, Leverage, Coverage and Activity

(c) HOCK international, page 48


Part 2 : 11/10/17 09:52:04

A. A firm that is less highly leveraged has lower fixed costs. Because fixed costs are lower, profits as a
percentage of sales fluctuate less as the level of sales changes than would be the case for a more highly
leveraged firm. This will lead to less volatile, more stable earnings per share.

B. The more highly leveraged a firm is, the more volatile its earnings per share will be. This is because it has more
fixed costs which will cause profits as a percentage of sales to fluctuate more as sales fluctuate.

C. Whether or not a less highly leveraged firm will have higher or lower earnings per share depends on whether or not
they are operating above or below the breakeven point.

D. Whether or not a more highly leveraged firm will have higher or lower earnings per share depends on whether or not
it is operating above or below the breakeven point.

Question 23 - CMA 688 4.15 - Ratios: Liquidity, Leverage, Coverage and Activity

A. The use of the calendar year for the accounting period will not always understate the ratio.

B. Not using average receivables in the days' sales-in-receivables ratio will not always understate the ratio.

C.

The days' sales-in-receivables ratio is calculated as the days in the year divided by the receivables turnover
ratio. The receivables turnover ratio is calculated as sales divided by average receivables. In order to
understate the days' sales-in-receivables ratio, either the sales need to be understated or the receivables
turnover ratio needs to be overstated.

A natural business year ends at the end of its main selling season when inventories have been depleted and
thus are low and accounts receivable from the selling season have been collected and thus are also low. If the
company uses the natural business year for its accounting period, the accounts receivable balances will be
understated because the natural business year will start and end at the down cycle of the business. Because
the receivables will be understated, the receivables turnover will be higher than it would be otherwise. This
will lead to an understated days' sales-in-receivables ratio.

D. The use of average receivables in the days' sales-in-receivables ratio will not always understate the ratio.

Question 24 - ICMA 10.P2.025 - Ratios: Liquidity, Leverage, Coverage and Activity

A.

The current ratio is current assets divided by current liabilities. When the current ratio is greater than 1, it means the
company's current assets are greater than its current liabilities.

When current assets are greater than current liabilities, and both current assets and current liabilities increase by the
same amount, the percentage of increase in current assets is less than the percentage of increase in current liabilities.
Since the percentage of increase for current assets was lower than the percentage of increase for current liabilities, the
current ratio declined as a result of the inventory purchase.

B.

The easiest way to see how and why this works the way it does is to make up some numbers. Since the
current ratio now is 2.3, let's say that current assets now are $2,300,000 and current liabilities are $1,000,000.

(c) HOCK international, page 49


Part 2 : 11/10/17 09:52:04

(NOTE: Total assets are given as $7,500,000, but total assets is meaningless in calculating the current ratio.
The current ratio is current assets divided by current liabilities. Total assets cannot be used to calculate the
current ratio.)

If Davis purchases $750,000 of inventory, inventory will increase by $750,000 and current assets will increase
from $2,300,000 to $3,050,000. If the purchase is made on credit, accounts payable will increase by $750,000,
and total current liabilities will increase from $1,000,000 to $1,750,000. The current ratio will become
$3,050,000 ÷ $1,750,000, which equals 1.74. That is lower than 2.3.

This occurred because before the inventory purchase, current assets were larger than current liabilities (we
know that because the current ratio was greater than 1.00). Current assets and current liabilities both
increased by the same amount. But for current assets, that amount was a lower percentage of increase than it
was for current liabilities, since current assets were higher than current liabilities. Since the percentage of
increase for current assets was lower than the percentage of increase for current liabilities, the current ratio
declined as a result of the inventory purchase.

C.

The current ratio is current assets divided by current liabilities. When the current ratio is greater than 1, it means the
company's current assets are greater than its current liabilities.

When current assets are greater than current liabilities, and both current assets and current liabilities increase by the
same amount, the percentage of increase in current assets is less than the percentage of increase in current liabilities.
Since the percentage of increase for current assets was lower than the percentage of increase for current liabilities, the
current ratio declined as a result of the inventory purchase.

D.

The current ratio is current assets divided by current liabilities. When the current ratio is greater than 1, it means the
company's current assets are greater than its current liabilities.

When current assets are greater than current liabilities, and both current assets and current liabilities increase by the
same amount, the percentage of increase in current assets is less than the percentage of increase in current liabilities.
Since the percentage of increase for current assets was lower than the percentage of increase for current liabilities, the
current ratio declined as a result of the inventory purchase.

Question 25 - CMA 687 4.27 - Ratios: Liquidity, Leverage, Coverage and Activity

A. There is a relationship between the magnitude of a company's debt-to-equity ratio and the magnitude of the same
company' debt-to-assets ratio. See the correct answer for a complete explanation.

B. These two ratios could not be the same because the assets and equity of a company will be different from each
other (except in the unusual situation in which the company has no debt).

C. Debt to equity is total liabilities divided by total equity. Debt to assets is total liabilities divided by total
assets. Because the asset balance of a company is higher than its equity, a debt-to-equity ratio will be higher
than a debt-to-asset ratio because when equity is used in the denominator, the denominator will be smaller
than when assets are used in the denominator. We know that the assets of the company must be higher than
the equity of the company because of the accounting equation of assets = liabilities + equity.

D. Debt to equity is total liabilities divided by total equity. Debt to assets is total liabilities divided by total assets.
Because the asset balance of a company is higher than its equity, a debt-to-equity ratio will be higher than a
debt-to-asset ratio because when equity is used in the denominator, the denominator will be smaller than when assets
are used in the denominator.

(c) HOCK international, page 50


Part 2 : 11/10/17 09:52:04

Question 26 - ICMA 10.P2.021 - Ratios: Liquidity, Leverage, Coverage and Activity

A. Working capital (or net working capital) is total current assets minus total current liabilities. This answer identifies the
current assets and current liabilities correctly. However, it results from including available-for-sale securities at their
historical cost. Available-for-sale securities are reported at their current market value in the financial statements. Thus,
they should be included at their fair value at year end.

B. Working capital (or net working capital) is total current assets minus total current liabilities. Current assets
include Cash ($10,000), Accounts Receivable ($20,000), Prepaids ($8,000), Inventory ($30,000) and
available-for-sale securities at their fair value at year end ($12,000), for a total of $80,000. Current liabilities
include Accounts Payable ($15,000) and Notes Payable due in 90 days ($25,000), for a total of $40,000. Thus,
working capital is $80,000 – $40,000, or $40,000.

C. Working capital (or net working capital) is total current assets minus total current liabilities. This answer omits
inventory from total current assets. Inventory is a current asset and should be included.

D. Working capital (or net working capital) is total current assets minus total current liabilities. This answer omits
available-for-sale securities from total current assets. Available-for-sale securities can be classified as either current or
non-current assets, depending upon management's intentions for them; however, the problem states that here, they
are classified as current assets.

Question 27 - ICMA 13.P2.004 - Ratios: Liquidity, Leverage, Coverage and Activity

A.

DOL is calculated as either [% change in EBIT / % change in Sales] or as [Contribution margin / Operating income].
This answer does not calculate DOL correctly.

We have been unable to determine how to calculate this incorrect answer choice. If you have calculated it, please let
us know how you did it&nbsp;so we can create a full explanation of why this answer choice is incorrect. Please send
us&nbsp;an email at support@hockinternational.com. Include the full Question ID number and the actual incorrect
answer choice -- not its letter, because that can change with every study session created. The Question ID number
appears in the upper right corner of the ExamSuccess screen. Thank you in advance for helping us to make your
HOCK study materials better.

B. DOL can be calculated as [% change in EBIT / % change in Sales]. It can also be calculated as [Contribution
Margin / Operating Income]. However, the financial statements in this problem are not structured to present a
contribution margin. Therefore, the only way to calculate DOL is [% change in EBIT / % change in Sales]. The
% change in EBIT is ($960,000 − $800,000) / $800,000, which is equal to 0.20. The % change in Sales is
($4,400,000 − $4,000,000) / $4,000,000 which is 0.10. DOL is equal to 2.00 calculated as 0.20 / 0.10.

C. DOL is calculated as [% change in EBIT / % change in Sales]. This answer uses total operating costs in the
numerator instead of earnings before interest and taxes (EBIT).

D. DOL is calculated as [% change in EBIT / % change in Sales]. This uses net income instead of earnings before
interest and taxes (EBIT) as the numerator.

Question 28 - ICMA 10.P2.044 - Ratios: Liquidity, Leverage, Coverage and Activity

A.

(c) HOCK international, page 51


Part 2 : 11/10/17 09:52:04

This is not the correct answer. Please see the correct answer for an explanation.

We have been unable to determine how to calculate this incorrect answer choice. If you have calculated it, please let
us know how you did it so we can create a full explanation of why this answer choice is incorrect. Please send us an
email at support@hockinternational.com. Include the full Question ID number and the actual incorrect answer choice --
not its letter, because that can change with every study session created. The Question ID number appears in the upper
right corner of the ExamSuccess screen. Thank you in advance for helping us to make your HOCK study materials
better.

B. This is December 31 long-term debt ($77,000) divided by common stock ($300,000). The debt/equity ratio is total
liabilities divided by total equity.

C. This is total liabilities divided by common stock + retained earnings. The reserve for bond retirement is an
appropriation of retained earnings and is an equity account, as well.

D.

The debt/equity ratio is total liabilities divided by total equity. When calculating a ratio that consists of all
balance sheet amounts, such as the debt/equity ratio, we do not use averages. We use the most current
period end figures that are available. Here, that is the December 31 balances.

The reserve for bond retirement account is an appropriation of retained earnings. All retained earnings start
out classified as Unappropriated Retained Earnings. The term "unappropriated" simply means that the
retained earnings are available to be distributed to shareholders in the form of dividends. Occasionally,
however, a company does not want to distribute a portion of its retained earnings, and this can be
communicated to the shareholders (and potential shareholders) through the process of appropriating a
portion of retained earnings.

The appropriation of retained earnings is done by the board of directors, and there is only one result of this
action. This action informs the readers of the financial statements that some of the retained earnings are not
available for distribution. There is no legal meaning to this, no time period involved (the board can
de-appropriate the retained earnings at any time), and there are no involved accounting processes to this.

There are many reasons that a company may decide to appropriate retained earnings. Among them are:
creating a reserve to build a plant, meeting the requirements of a bond, or simply providing for the future.
Remember, however, that the only effect of appropriating retained earnings is to let shareholders know that
the appropriated retained earnings will not be distributed as a dividend.

So the reserve for bond retirement is a line item in the equity section of the balance sheet and it is treated just
like retained earnings in the calculation of the debt/equity ratio.

As of December 31, Total Liabilities is $172,000 ($84,000 + $11,000 + $77,000) and Total Equity is $534,000
($300,000 + $28,000 + $206,000). The debt/equity ratio is therefore $172,000 ÷ $534,000, which equals 32.2%.

Question 29 - ICMA 10.P2.041 - Ratios: Liquidity, Leverage, Coverage and Activity

A. Leverage generally refers to the ability to receive a high level of return relative tothe amount of cost expended.
Financial leverage is the use of debt to increase earnings. The cost of using debt to finance operations is interest.
Interest is a fixed charge because unlike dividends, interest must be paid whether the firm is profitable or not. The use
of financing that carries a fixed charge is called financial leverage. The use of debt in the capital structure of a firm
changes its financial leverage, but it does not decrease its financial leverage.

B. Leverage generally refers to the ability to receive a high level of return relative tothe amount of cost expended.
Operating leverage measures the use of operating fixed cost to generate greater operating profit. Operating leverage

(c) HOCK international, page 52


Part 2 : 11/10/17 09:52:04

refers to the fact that, for a given level of fixed expenses, a given percentage change in sales will result in a higher
percentage of change in profits than the percentage change in sales. The use of debt in the capital structure of a firm
does not decrease its operating leverage, because the cost of debt is not an operating fixed cost.

C. Leverage generally refers to the ability to receive a high level of return relative tothe amount of cost expended.
Operating leverage measures the use of operating fixed cost to generate greater operating profit. Operating leverage
refers to the fact that, for a given level of fixed expenses, a given percentage change in sales will result in a higher
percentage of change in profits than the percentage change in sales. The use of debt in the capital structure of a firm
does not increase its operating leverage, because the cost of debt is not an operating fixed cost.

D. Leverage generally refers to the ability to receive a high level of return relative to the amount of cost
expended. Financial leverage is the use of debt to increase earnings. The cost of using debt to finance
operations is interest. Interest is a fixed charge because unlike dividends, interest must be paid whether the
firm is profitable or not. The use of financing that carries a fixed charge is called financial leverage. The use of
debt in the capital structure of a firm increases its financial leverage.

Question 30 - CMA 692 2.27 - Ratios: Liquidity, Leverage, Coverage and Activity

A. If the company changes from FIFO to LIFO during a period of rising prices, its ending inventory value will decrease.
Under LIFO the ending inventory is made up of the oldest items in inventory and these will be the less expensive items
in a period of rising prices. Also, the cost of goods sold will increase because the company is assuming that the newer,
more expensive items in inventory are the ones sold. Inventory turnover is calculated as cost of goods sold divided by
the average inventory. Since this change to LIFO would increase the numerator and decrease the denominator, this
ratio will increase as a result of the change.

B. The cash flows of the company are not affected by the method of inventory tracking.

C. If the company changes from FIFO to LIFO during a period of rising prices, its ending inventory value will
decrease. Under LIFO the ending inventory is made up of the oldest items in inventory and these will be the
less expensive items in a period of rising prices. Also, the cost of goods sold will increase because the
company is assuming that the newer, more expensive items in inventory are the ones sold. Since inventory is
part of the numerator of the current ratio and it is now smaller, the current ratio for the company would be
reduced if the company switched from FIFO to LIFO.

D. The debt-to-equity ratio is not directly affected by the method of inventory tracking that is used. However, it is
affected through a higher or lower profit from the sale of inventory. If the company switched to LIFO, they would have a
higher cost of goods sold and this will decrease profits. The lower profits will decrease equity, which would increase the
debt-to-equity ratio.

Question 31 - ICMA 13.P2.003 - Ratios: Liquidity, Leverage, Coverage and Activity

A.

The cash ratio is

Cash + Cash Equivalents + Marketable Securities

Current Liabilities

If the company purchases materials on account, current liabilities will increase. The numerator of the ratio will
remain the same while the denominator increases, which will cause the ratio to decrease.

(c) HOCK international, page 53


Part 2 : 11/10/17 09:52:04

B.

The cash ratio is

Cash + Cash Equivalents + Marketable Securities

Current Liabilities

The numerator and the denominator of the ratio will increase by equal amounts as a result of the short-term loan.
Whether the cash ratio would increase or decrease depends upon whether the numerator or the denominator of the
ratio had been greater before the money was borrowed.

If the numerator had been greater than the denominator, an equal increase in the numerator and denominator will
cause a decrease in the cash ratio. For example, if the numerator were 100 and the denominator were 75, the cash
ratio would be 1.33. If the amount of the loan were 20, the numerator would increase to 120 and the denominator
would increase to 95. The cash ratio would decrease to 1.26.

If the denominator had been greater than the numerator, an equal increase in the numerator and denominator will
cause an increase in the cash ratio. If the numerator were 75 and the denominator were 100, the cash ratio would be
0.75. An equal increase of 20 would cause the numerator to increase to 95, the denominator to increase to 120, and
the cash ratio to increase to 0.79.

This occurs because an equal increase in the numerator and the denominator of any ratio will represent a smaller
proportional increase to the larger number than to the smaller number. If the larger number is in the numerator, it will
cause the ratio to decrease. If the larger number is in the denominator, it will cause the ratio to increase.

C.

The cash ratio is

Cash + Cash Equivalents + Marketable Securities

Current Liabilities

Commercial paper is a cash equivalent. If the company purchases commercial paper with cash, the purchase will
cause a decrease in cash and an equal increase in cash equivalents. Neither the numerator nor the denominator of the
cash ratio will change and thus the cash ratio will not change.

D.

The cash ratio is

Cash + Cash Equivalents + Marketable Securities

Current Liabilities

Selling goods for cash will cause the numerator of the ratio to increase because cash will increase. That will be true
whether the goods are sold at a price lower than cost or higher than cost. The denominator will not change as a result
of the sale. Thus the cash ratio will increase, not decrease.

Question 32 - ICMA 10.P2.033 - Ratios: Liquidity, Leverage, Coverage and Activity

(c) HOCK international, page 54


Part 2 : 11/10/17 09:52:04

A. The acid test, or quick, ratio is Cash + Marketable Securities + Accounts Receivable divided by Current Liabilities.
Since cash is included in the numerator of the ratio, it cannot indicate the company's ability to pay its current liabilities
without having to reduce its cash balance. Furthermore, current liabilities cannot be paid without reducing cash.

B. The acid test, or quick, ratio is Cash + Marketable Securities + Accounts Receivable divided by Current Liabilities.
Since accounts receivable are included in the numerator of the ratio, it cannot indicate the company's ability to pay its
current liabilities without having to collect its receivables. It is assumed that the receivables will be collected and the
cash used to pay the current liabilities.

C. The acid test, or quick, ratio is Cash + Marketable Securities + Accounts Receivable divided by Current Liabilities. It
is the most conservative ratio of the liquidity ratios, because it does not include Inventory in the numerator. It does not
indicate the company's ability to pay its current liabilities without having to borrow additional funds. Borrowing additional
funds should probably not be the first choice of a company that is unable to pay its current liabilities. If borrowing were
short-term, the borrowings would increase current liabilities, further reducing the acid test ratio.

D. The acid test, or quick, ratio is Cash + Marketable Securities + Accounts Receivable divided by Current
Liabilities. Since inventory is not included in the numerator, the ratio measures whether the firm has adequate
current assets to cover its current liabilities without having to liquidate any of its inventory.

Question 33 - CMA 1294 2.22 - Ratios: Liquidity, Leverage, Coverage and Activity

A. The average number of days to collect receivables is calculated as 365 divided by the receivables turnover
ratio. Receivables turnover is calculated as net annual credit sales divided by average receivables. Credit
sales were $6,205,000 and average receivables were $335,000 (the average of the beginning and ending
balances). This gives a receivables turnover ratio of 18.52. Dividing 365 by 18.52 gives us 19.71 days as the
number of days to collect receivables.

B. The average number of days to collect receivables is calculated as 365 divided by the receivables turnover ratio.
Receivables turnover is calculated as net annual credit sales divided by average receivables.

C. The average number of days to collect receivables is calculated as 365 divided by the receivables turnover.
Receivables turnover is calculated as net annual credit sales divided by average receivables.

D. The average number of days to collect receivables is calculated as 365 divided by the receivables turnover ratio.
Receivables turnover is calculated as net annual credit sales divided by average receivables.

Question 34 - ICMA 13.P2.015 - Ratios: Liquidity, Leverage, Coverage and Activity

A.

Lawrence Inc. uses the percentage of sales method to estimate its bad debts expense. The percentage of
sales method focuses on the amount of sales made during the period and on determining what percentage of
those sales will not be collectible.

Lawrence has increased its estimate of bad debt expense from 3% of its $25,000,000 in sales to 4.5% of sales.
Therefore, bad debt expense will increase by the amount of the difference, or 1.5% of $25,000,000, which is
$375,000.

Degree of operating leverage is calculated as Contribution Margin divided by Operating Income. The best way
to evaluate this is to make up some numbers for the contribution income statement and evaluate it using 3%
of sales and 4.5% of sales for bad debt expense. Because bad debt expense is a percentage of sales, it is a
variable expense. Therefore, the contribution income statements using 3% and 4.5% for bad debt expense

(c) HOCK international, page 55


Part 2 : 11/10/17 09:52:04

would be as follows:
Using 3% Using 4.5%
Sales $25,000,000 $25,000,000
Variable costs 15,000,000 15,375,000
Contribution margin $10,000,000 $9,625,000
Fixed costs 7,000,000 7,000,000
Operating income $3,000,000 $2,625,000

Degree of operating leverage using 3% of sales for bad debt expense would be $10,000,000 ÷ $3,000,000, or
3.33. Degree of operating leverage using 4.5% of sales for bad debt expense would be $9,625,000 ÷ $2,625,000,
or 3.67, an increase.

B.

Lawrence Inc. uses the percentage of sales method to estimate its bad debts expense. $30,000 is the difference
between 3% of ending accounts receivable and 4.5% of ending accounts receivable. The percentage of sales method
focuses on the amount of sales made during the period and on determining what percentage of those sales will not be
collectible, not on a percentage of receivables.

Lawrence has increased its estimate of bad debt expense from 3% of its $25,000,000 in sales to 4.5% of sales.
Therefore, bad debt expense will increase by the amount of the difference, or 1.5% of $25,000,000.

The financial leverage ratio is Total Assets divided by Common Equity. An increase in bad debt expense from 3% to
4.5% of sales would decrease total assets (by decreasing net accounts receivable) and common equity (by decreasing
retained earnings) by the same amount. Since common equity is a smaller absolute amount than total assets, this
decrease would be a proportionately larger decrease for common equity than for assets. The denominator of the
financial leverage ratio would decrease proportionately more than the numerator. The result would be an increase in
the financial leverage ratio, not a decrease.

C. The degree of operating leverage will not be decreased. Please see the correct answer explanation for the reason.
The question does not give information that would enable calculation of times interest earned.

D.

$1,125,000 is the amount of bad debt expense as 4.5% of sales. However, the decrease in the current year's income
is the amount of increase in the bad debt expense (from 3% to 4.5% of sales), not the amount of bad debt expense.

The degree of operating leverage will not be decreased. Please see the correct answer explanation for the reason.

Question 35 - CIA 1193 IV.46 - Ratios: Liquidity, Leverage, Coverage and Activity

A. The quick ratio is calculated as follows: [(cash + receivables+ trading securities) ÷ current liabilities]. The
current liabilities include accounts payable ($67,000), income taxes payable ($70,000), notes payable within 1
year ($54,000) and other current liabilities ($22,500). These total $213,500. The current assets included in the
quick ratio are cash ($27,500), receivables ($115,000) and trading securities ($64,000). These total $206,500
giving a quick ratio of 0.97 ($206,500 ÷ $213,500).

B. This answer includes prepaid expenses as a current asset. See the correct answer for a complete explanation.

C. This answer includes deferred tax liabilities-noncurrent as a current liability. See the correct answer for a complete
explanation.

D. This answer does not include income taxes payable as a current liability. See the correct answer for a complete
explanation.

(c) HOCK international, page 56


Part 2 : 11/10/17 09:52:04

Question 36 - ICMA 10.P2.059 - Ratios: Liquidity, Leverage, Coverage and Activity

A. The accounts receivable turnover ratio is net annual credit sales / average accounts receivable. Net annual
accounts receivable is total sales of $1,700,000 × (1 − 0.06), or $1,598,000. Average accounts receivable is the
average of the beginning and ending accounts receivable balances, which is ($168,000 + $147,000) ÷ 2, or
$157,500. The accounts receivable turnover ratio is therefore $1,598,000 ÷ $157,500, which equals 10.15.

B. The accounts receivable turnover ratio is net annual credit sales / average accounts receivable. This is total sales /
average accounts receivable. Since 6% of the company's sales are for cash, net credit sales are 94% (1 − 6%) of total
sales.

C. The accounts receivable turnover ratio is net annual credit sales ÷ average accounts receivable. This is net annual
credit salaes ÷ beginning accounts receivable.

D. The accounts receivable turnover ratio is net annual credit sales ÷ average accounts receivable. This is net annual
credit salaes ÷ ending accounts receivable.

Question 37 - ICMA 10.P2.046 - Ratios: Liquidity, Leverage, Coverage and Activity

A. The current ratio is a measure of short-term liquidity, as it relates current assets to the claims of short-term
creditors. It is not an indicator of the firm's long-term debt paying ability.

B.

Sources disagree on how the working capital ratio is calculated and what it means. Fortunately, it is not a ratio that is
tested on the CMA exam. Here are two different definitions:
1. The working capital turnover ratio is Net Sales ÷ Average Net Working Capital. It provides a measure to
compare the depletion of working capital to the generation of sales over a specific period of time. It indicates
how effectively the company is using its working capital to generate sales. An increasing working capital turnover
ratio indicates either that the company is achieving greater sales with the same or a lesser amount of working
capital, or it has been able to maintain the same level of sales with reduced working capital.
2. The working capital turnover ratio is Cost of Sales ÷ Average Net Working Capital. It represents the number of
times the working capital is turned over in the course of year and indicates the velocity of the utilization of net
working capital.

Regardless of which definition and interpretation is used, the working capital turnover ratio is not an indicator of the
firm's long-term debt paying ability.

C. The Debt to Total Assets Ratio measures the proportion of the company's total assets that are financed by
creditors and thus the firm’s long-term debt repayment ability. Creditors would like this ratio to be as low as
possible because it indicates a lower chance of default on the interest payments that the company will owe.
Therefore, the higher this ratio is, the higher the company's cost of debt will be, because creditors will
demand compensation for the increased risk they are bearing.

D. The Asset Turnover Ratio measures the amount of sales revenue the company is generating from the use of all of
its assets. It provides a means to measure the overall efficiency of the company’s use of all of its investments, as
represented by both short-term assets and long-term assets. It is not an indicator of the firm's long-term debt paying
ability.

(c) HOCK international, page 57


Part 2 : 11/10/17 09:52:04

Question 38 - CMA 1280 4.2 - Ratios: Liquidity, Leverage, Coverage and Activity

A. Because Depoole has a current ratio of greater than 1.0 (since total quick assets exceeded total current
liabilities), the purchase of raw materials on account will decrease the current ratio. This is because the
purchase of materials on account will increase both the current assets and the current liabilities by the same
amount. Let us assume that Depoole had current assets of $150 and current liabilities of $100, giving a current
ratio of 1.50. Now assume that they purchased $50 of materials. This increased the current assets to $200 and
the current liabilities to $150. Now, the ratio is 1.33, which is lower than it had been.

B. Because this transaction increases current assets and current liabilities by the same amount, there is no change in
the level of working capital.

C. Because this transaction increases current assets and current liabilities by the same amount, there is no change in
the level of working capital.

D. Because the current ratio was greater than 1.0, this transaction will decrease the current ratio. See the correct
answer for a complete explanation.

Question 39 - CMA 679 4.13 - Ratios: Liquidity, Leverage, Coverage and Activity

A. When stock options issued by the company to officers are exercised there are more shares outstanding. This will
decrease the ownership interest of existing shareholders because the number of shares that they own is now a smaller
percentage of the larger total number of shares outstanding. A decrease in the ownership interest of existing
shareholders is not an improvement.

B. When stock options issued by the company to officers are exercised there are more shares outstanding. This will
decrease the basic earnings per share because the same amount of income is being divided among a larger number
of shares. A decrease in basic earnings per share is not an improvement.

C. Total asset turnover is calculated as sales divided by total assets. When stock options issued by the company to
officers are exercised the company will receive some amount of cash. This will increase total assets and decrease the
total asset turnover, which is not an improvement.

D. The debt-to-equity ratio is calculated as total debt (i.e., total liabilities) divided by total equity. The exercise
of the options will increase total equity and this will decrease the debt-to-equity ratio. A lower debt-to-equity
ratio is an improvement.

Question 40 - ICMA 10.P2.052 - Ratios: Liquidity, Leverage, Coverage and Activity

A.

Since it asks for the number of days, this question is asking for the number of days' sales in accounts
receivable, not the accounts receivable turnover ratio. The number of days' sales in accounts receivable can
be calculated in either of two ways:

(1) Calculate the accounts receivable turnover ratio (the number of times accounts receivable "turns over"
during a year's time), then divide the number of days in the year by the accounts receivable turnover ratio, as
follows:

Accounts receivable turnover ratio = Net credit sales for the period ÷ Average accounts receivable.

Average accounts receivable is the average of the beginning and ending balances, which is $22,000. Thus, the

(c) HOCK international, page 58


Part 2 : 11/10/17 09:52:04

accounts receivable turnover ratio is $220,000 ÷ $22,000 = 10 times.

The number of days' sales in accounts receivable is 365 ÷ 10, which equals 36.5 days.

(2) Calculate the amount of credit sales made per day, then divide average accounts receivable by the credit
sales made per day, as follows:

Net credit sales for the year ÷ number of days in the year = credit sales made per day. Average accounts
receivable ÷ credit sales made per day = number of days' sales in accounts receivable.

$220,000 ÷ 365 = daily credit sales of $602.74. The number of days' sales in accounts receivable = Average
accounts receivable of $22,000 ÷ $602.74, which equals 36.5 days.

B.

This is not the correct answer. Please see the correct answer for an explanation.

We have been unable to determine how to calculate this incorrect answer choice. If you have calculated it, please let
us know how you did it so we can create a full explanation of why this answer choice is incorrect. Please send us an
email at support@hockinternational.com. Include the full Question ID number and the actual incorrect answer choice --
not its letter, because that can change with every study session created. The Question ID number appears in the upper
right corner of the ExamSuccess screen. Thank you in advance for helping us to make your HOCK study materials
better.

C.

This is the days' sales in accounts receivable calculated using the year end accounts receivable balance. The number
of days' sales in accounts receivable should be calculated using the average balance in accounts receivable instead.
The average balance is the average of the beginning and ending balances.

Whenever we relate an income statement amount such as net credit sales to a balance sheet amount such as
accounts receivable, we need to use the average balance of the balance sheet amount, not the beginning or ending
balance.

D.

This is the days' sales in accounts receivable calculated using the beginning of the year accounts receivable balance.
The number of days' sales in accounts receivable should be calculated using the average balance in accounts
receivable instead. The average balance is the average of the beginning and ending balances.

Whenever we relate an income statement amount such as net credit sales to a balance sheet amount such as
accounts receivable, we need to use the average balance of the balance sheet amount, not the beginning or ending
balance.

Question 41 - CIA 1195 IV.36 - Ratios: Liquidity, Leverage, Coverage and Activity

A. The current ratio is one of the ratios used to measure the liquidity of a company.

B. Profit margin on sales is a profitability measure.

C. Total debt to total assets ratio is a measure of the use of debt in the company.

D. The days sales outstanding ratio measures how quickly receivables are collected. This is part of liquidity, but it is not
a measure of liquidity.

(c) HOCK international, page 59


Part 2 : 11/10/17 09:52:04

Question 42 - ICMA 10.P2.058 - Ratios: Liquidity, Leverage, Coverage and Activity

A.

The current ratio is Current Assets ÷ Current Liabilities. If the company changes from FIFO to LIFO during a period of
inflation, inventory will decrease because the inventory on the balance sheet will consist of units purchased first, when
the costs were lower. When Inventory decreases, Current Assets will decrease as well. If the numerator of the current
ratio decreases while the denominator stays the same, the Current Ratio will decrease.

The inventory turnover ratio is Annual COGS ÷ Average Inventory. If the company changes from FIFO to LIFO during a
period of inflation, inventory will decrease and COGS will increase. Inventory will decrease because the inventory on
the balance sheet will consist of units purchased or manufactured first, when the costs were lower. COGS will increase
because the units sold will be the last units purchased, when costs were higher. If the numerator of the inventory
turnover ratio increases and the denominator decreases, the inventory turnover ratio will increase.

B.

The current ratio is Current Assets ÷ Current Liabilities. If the company changes from FIFO to LIFO during a period of
inflation, inventory will decrease because the inventory on the balance sheet will consist of units purchased first, when
the costs were lower. When Inventory decreases, Current Assets will decrease as well. If the numerator of the current
ratio decreases while the denominator stays the same, the Current Ratio will decrease.

The inventory turnover ratio is Annual COGS ÷ Average Inventory. If the company changes from FIFO to LIFO during a
period of inflation, inventory will decrease and COGS will increase. Inventory will decrease because the inventory on
the balance sheet will consist of units purchased or manufactured first, when the costs were lower. COGS will increase
because the units sold will be the last units purchased, when costs were higher. If the numerator of the inventory
turnover ratio increases and the denominator decreases, the inventory turnover ratio will increase.

C.

The current ratio is Current Assets ÷ Current Liabilities. If the company changes from FIFO to LIFO during a period of
inflation, inventory will decrease because the inventory on the balance sheet will consist of units purchased first, when
the costs were lower. When Inventory decreases, Current Assets will decrease as well. If the numerator of the current
ratio decreases while the denominator stays the same, the Current Ratio will decrease.

The inventory turnover ratio is Annual COGS ÷ Average Inventory. If the company changes from FIFO to LIFO during a
period of inflation, inventory will decrease and COGS will increase. Inventory will decrease because the inventory on
the balance sheet will consist of units purchased or manufactured first, when the costs were lower. COGS will increase
because the units sold will be the last units purchased, when costs were higher. If the numerator of the inventory
turnover ratio increases and the denominator decreases, the inventory turnover ratio will increase.

D.

The current ratio is Current Assets ÷ Current Liabilities. If the company changes from FIFO to LIFO during a
period of inflation, inventory will decrease because the inventory on the balance sheet will consist of units
purchased first, when the costs were lower. When Inventory decreases, Current Assets will decrease as well.
If the numerator of the current ratio decreases while the denominator stays the same, the Current Ratio will
decrease.

The inventory turnover ratio is Annual COGS ÷ Average Inventory. If the company changes from FIFO to LIFO
during a period of inflation, inventory will decrease and COGS will increase. Inventory will decrease because
the inventory on the balance sheet will consist of units purchased or manufactured first, when the costs were
lower. COGS will increase because the units sold will be the last units purchased, when costs were higher. If
the numerator of the inventory turnover ratio increases and the denominator decreases, the inventory
turnover ratio will increase.

(c) HOCK international, page 60


Part 2 : 11/10/17 09:52:04

Question 43 - ICMA 13.P2.029 - Ratios: Liquidity, Leverage, Coverage and Activity

A. Changing the amount of the accrued vacation liability will not impact the amount of cash the company has.
Decreasing the accrued vacation liability will increase net income, but not cash.

B. A reduction of the number of days of sales outstanding in accounts receivable means that we are collecting
our receivables faster. This will decrease accounts receivable and increase the cash balance from one period
to the next because there will be fewer days of sales outstanding in accounts receivable than there were in the
previous period.

C. Paying our payables faster will mean that in the current period we paid for more total days of purchases than we had
done in the past. This will reduce the amount of cash from the previous period.

D. Holding more inventory will require a greater cash investment for that inventory. This will reduce the amount of cash
we hold compared to the previous period, not increase it.

Question 44 - CMA 692 1.10 - Ratios: Liquidity, Leverage, Coverage and Activity

A.

The degree of total leverage is equal to the degree of operating leverage times the degree of financial leverage. Thus,
a decrease in either of these ratios results in a proportional decrease in total leverage.

B. The degree of total leverage is equal to the degree of operating leverage times the degree of financial
leverage. Thus, a decrease in either of these ratios results in a decrease in total leverage. If the company had
no preferred stock, the DFL and the DTL would be lower because the pretax income necessary to pay the
preferred dividends [P / (1 − T)] is subtracted from the denominator of the DFL.

C.

The degree of total leverage is equal to the degree of operating leverage times the degree of financial leverage. Thus,
an increase in either of these ratios results in an increase in total leverage. If the company had no preferred stock, the
DFL and the DTL would be lower, not higher, because the pretax income necessary to pay the preferred dividends [P /
(1 − T)] is subtracted from the denominator of the DFL. If the preferred dividends were not subtracted from the
denominator, the denominator would be higher and the DFL would be lower.

Because the increases would not be proportional.

D. The elimination of preferred stock would change both DFL and DTL. The pretax income necessary to pay the
preferred dividends [P / (1 − T)] is subtracted from the denominator of the DFL. If the preferred dividends were not
subtracted from the denominator, the denominator would change and the DFL would change.

Question 45 - CMA 1280 4.7 - Ratios: Liquidity, Leverage, Coverage and Activity

A. Both the current and the quick ratios will be affected by this transaction because this transaction reduces the current
and quick assets used in the calculation of these ratios.

B. While both of these ratios will decrease as a result of this transaction, the current ratio will not be affected by a

(c) HOCK international, page 61


Part 2 : 11/10/17 09:52:04

greater degree than the quick ratio. This is because the numerator in the current and quick ratios are different. The
numerator in the current ratio includes inventory, but the numerator in the quick ratio does not include inventory.
Therefore, the quick ratio will be affected by a greater degree than the current ratio.

C.

The early liquidation of a long-term note with cash will reduce the level of current and quick assets but will not
affect the level of current liabilities. Therefore, both the current and quick ratios will decrease as a result of
this transaction. However, the degree of the reduction in the quick ratio will be greater than the degree of the
reduction in the current ratio because the numerator in the quick ratio is smaller. This is because the quick
ratio includes only cash, marketable securities and accounts receivable in the numerator. It does not include
inventory or other current assets such as prepaids in the numerator.

Degree of reduction means percentage of reduction, not an absolute amount of reduction.

D. While both of these ratios will decrease as a result of this transaction, they will not be affected equally. This is
because the numerator in the current and quick ratios are different. The numerator in the current ratio includes
inventory, but the numerator in the quick ratio does not include inventory.

Question 46 - ICMA 10.P2.038 - Ratios: Liquidity, Leverage, Coverage and Activity

A. Degree of Financial Leverage is EBIT ÷ EBT (Earnings Before Interest & Taxes ÷ Earnings Before Taxes).
EBIT is $15,000,000 − $9,000,000 − 3,000,000 = $3,000,000. EBT is EBIT − I, or $3,000,000 − 800,000, which
equals $2,200,000. Thus, Degree of Financial Leverage is $3,000,000 ÷ $2,200,000, which is 1.36.

B.

This is not the correct answer. Please see the correct answer for an explanation.

We have been unable to determine how to calculate this incorrect answer choice. If you have calculated it, please let
us know how you did it so we can create a full explanation of why this answer choice is incorrect. Please send us an
email at support@hockinternational.com. Include the full Question ID number and the actual incorrect answer choice --
not its letter, because that can change with every study session created. The Question ID number appears in the upper
right corner of the ExamSuccess screen. Thank you in advance for helping us to make your HOCK study materials
better.

C. Degree of Financial Leverage is EBIT ÷ EBT (Earnings Before Interest & Taxes ÷ Earnings Before Taxes). This is
(Net Income + Interest Expense) ÷ Net Income.

D. Degree of Financial Leverage is EBIT ÷ EBT (Earnings Before Interest & Taxes ÷ Earnings Before Taxes). This is
EBIT ÷ Net Income.

Question 47 - CMA 1291 1.5 - Ratios: Liquidity, Leverage, Coverage and Activity

A. Treasury stock is the corporation's own shares that it has repurchased on the market. The purchase of treasury
stock reduces owners' equity because those shares are no longer outstanding.

B. Treasury stock is the corporation's own shares that it has repurchased on the market. Treasury stock is not an
asset. The purchase of treasury stock reduces owners' equity because those shares are no longer outstanding. The
purchase of the treasury stock will reduce assets because cash will be reduced while reducing owners' equity.

C. Treasury stock is the corporation's own shares that it has repurchased on the market. The purchase of

(c) HOCK international, page 62


Part 2 : 11/10/17 09:52:04

treasury stock reduces owners' equity because those shares are no longer outstanding. The decrease in
equity causes the financial leverage of the company to increase.

D. The purchase of treasury shares does not affect the firm's interest-coverage ratio. Treasury stock is the
corporation's own shares that it has repurchased on the market. The purchase of treasury stock reduces owners'
equity because those shares are no longer outstanding. The interest coverage ratio is earnings before interest and
taxes divided by interest expense.

Question 48 - CMA 685 4.17 - Ratios: Liquidity, Leverage, Coverage and Activity

A. Return on equity is Net Income divided by Average Total Equity. In order to calculate the return on equity, we would
need to know the level of income of the company.

B. If the ratio of total liabilities to equity increases, the ratio of total liabilities to total assets must also
increase. Total liabilities plus total equity equals total assets. Therefore, if total liabilities become greater
relative to total equity, total liabilities must also become greater relative to total assets.

C. The current ratio is total current assets divided by total current liabilities. In order to determine the effect on the
current ratio we would need to know the change in current liabilities and current assets, neither of which are known
from the information given.

D. The times interest earned (interest coverage) ratio is Earnings Before Interest and Taxes divided by Interest
Expense. The ratio of total liabilities to equity will not affect the times interest earned ratio.

Question 49 - CMA 1280 4.6 - Ratios: Liquidity, Leverage, Coverage and Activity

A. All of the items listed in the other options will be affected in the manner in the choices. This makes this
choice the correct answer. Please see the other choices for complete explanations of the individual items.

B. Because the first installment of the bonds is due later this year, this will increase the current liabilities, thereby
decreasing the quick ratio. Because the bonds were issued in exchange for an office building, Depoole did not receive
any cash, so current assets did not increase. While it is correct that the quick ratio will decrease, this answer is not
correct because there is a better choice available.

C. Because the first installment of the bonds is due later this year, this will increase the current liabilities, thereby
decreasing net working capital. Because the bonds were issued in exchange for an office building, Depoole did not
receive any cash, so current assets did not increase. While it is correct that net working capital will decrease, this
answer is not correct because there is a better choice available.

D. Because the first installment of the bonds is due later this year, this will increase the current liabilities, thereby
decreasing the current ratio. Because the bonds were issued in exchange for an office building, Depoole did not
receive any cash, so current assets did not increase. While it is correct that the current ratio will decrease, this answer
is not correct because there is a better choice available.

Question 50 - ICMA 10.P2.031 - Ratios: Liquidity, Leverage, Coverage and Activity

A.

In answering this question, we must assume the question is referring to the credit manager of a business that is selling

(c) HOCK international, page 63


Part 2 : 11/10/17 09:52:04

to other businesses, and the credit manager is considering applications for short-term credit for purchases.

The ability of a firm to meet its short-term obligations is measured by its liquidity ratios. Profit margin and return on
assets are profitability ratios that indicate a firm's ability to meet its long-term obligations.

B.

In answering this question, we must assume the question is referring to the credit manager of a business that
is selling to other businesses, and the credit manager is considering applications for short-term credit for
purchases.

The ability of a firm to meet its short-term obligations is measured by its liquidity ratios. Liquidity ratios are
concerned with current assets and current liabilities. Working capital is current assets minus current
liabilities, and the current ratio is current assets divided by current liabilities. Thus, its working capital and its
current ratio are the best measurement of a credit applicant's ability to pay for goods and services received.

C.

In answering this question, we must assume the question is referring to the credit manager of a business that is selling
to other businesses, and the credit manager is considering applications for short-term credit for purchases.

The ability of a firm to meet its short-term obligations is measured by its liquidity ratios. The current ratio is a liquidity
ratio, but the price-earnings ratio − the price of a share of stock divided by the company's earnings per share − is not a
liquidity ratio.

D.

In answering this question, we must assume the question is referring to the credit manager of a business that is selling
to other businesses, and the credit manager is considering applications for short-term credit for purchases.

The ability of a firm to meet its short-term obligations is measured by its liquidity ratios. Working capital is a liquidity
ratio, but return on equity is not a liquidity ratio. Return on equity is a profitability ratio and is more a measure of the
firm's ability to meet its long-term obligations.

Question 51 - ICMA 10.P2.028 - Ratios: Liquidity, Leverage, Coverage and Activity

A.

The acid test, or quick, ratio is (Cash + Marketable Securities + Accounts Receivable) ÷ Total Current Liabilities. To
increase the ratio, Garstka will need to increase the numerator and/or decrease the denominator of the ratio.

Expediting collection of accounts receivable will not change the numerator because it will only transfer funds from
accounts receivable to cash, leaving the total of cash, marketable securities and accounts receivable the same. It also
will not change the denominator. So the acid test ratio will not change as a result of expediting collection of accounts
receivable.

B.

The acid test, or quick, ratio is (Cash + Marketable Securities + Accounts Receivable) ÷ Total Current
Liabilities. To increase the ratio, Garstka will need to increase the numerator and/or decrease the denominator
of the ratio.

Selling auto parts on account will increase the numerator because it will increase accounts receivable in the
numerator without changing the denominator. So the acid test ratio will increase as a result of selling auto
parts on account.

(c) HOCK international, page 64


Part 2 : 11/10/17 09:52:04

C.

The acid test, or quick, ratio is (Cash + Marketable Securities + Accounts Receivable) ÷ Total Current Liabilities. To
increase the ratio, Garstka will need to increase the numerator and/or decrease the denominator of the ratio.

Purchasing marketable securities for cash will not change the numerator, because Cash will decrease by the same
amount that Marketable Securities increases leaving the total of cash, marketable securities and accounts receivable
the same. The denominator will not change. So the acid test ratio will not change as a result of purchasing marketable
securities for cash.

D.

The acid test, or quick, ratio is (Cash + Marketable Securities + Accounts Receivable) ÷ Total Current Liabilities. To
increase the ratio, Garstka will need to increase the numerator and/or decrease the denominator of the ratio.

Making a payment to trade accounts payable will decrease the numerator and the denominator by the same amount,
because Cash and Accounts Payable will decrease by the same amount. What effect will that have on the acid test
ratio? Let's use some numbers and find out. Garstka's acid test ratio is 0.9. A 0.9 acid test ratio could result from a
numerator of $90,000 and a denominator of $100,000. Let's say a payment of $5,000 is made. Both the numerator and
the denominator are reduced by $5,000. The resulting acid test ratio is $85,000 ÷ $95,000, which is equal to 0.89. The
acid test ratio has gone down, not up.

The reason for this is because the beginning acid test ratio is less than 1.00. That means the beginning numerator is
lower than the denominator. When we decrease both the numerator and the denominator by the same dollar amount,
that amount is a greater percentage decrease for the numerator than it is for the denominator. A greater percentage
decrease for the numerator leads to a decrease in the ratio.

Question 52 - ICMA 10.P2.063 - Ratios: Liquidity, Leverage, Coverage and Activity

A.

The accounts payable turnover ratio is total annual credit purchases divided by average accounts payable. Purchases
for a reseller (retailer) can be approximated by adjusting cost of goods sold by the amount of change in inventory.
However, that cannot be used for a manufacturer. We are not told whether this company is a retailer or a
manufacturer, so it would not be a good idea to assume that it is a retailer. We need to work with what we have, and
the only thing we have to use is cost of goods sold.

This is cost of goods sold divided by the ending accounts payable, not the average accounts payable.

B.

The accounts payable turnover ratio is total annual credit purchases divided by average accounts payable. We can
calculate the average accounts payable from the information given as the average of the beginning and ending
accounts payable balances ([$3,320 + $3,680] ÷ 2 = $3,500). Purchases for a reseller (retailer) can be approximated
by adjusting cost of goods sold by the amount of change in inventory. However, that cannot be used for a
manufacturer. We are not told whether this company is a retailer or a manufacturer, so it would not be a good idea to
assume that it is a retailer. We need to work with what we have, and the only thing we have to use is cost of goods
sold. So we will use cost of goods sold in the place of purchases.

This is total credit sales divided by average accounts payable.

C.

The accounts payable turnover ratio is total annual credit purchases divided by average accounts payable. We
can calculate the average accounts payable from the information given as the average of the beginning and

(c) HOCK international, page 65


Part 2 : 11/10/17 09:52:04

ending accounts payable balances ([$3,320 + $3,680] ÷ 2 = $3,500). Purchases for a reseller (retailer) can be
approximated by adjusting cost of goods sold by the amount of change in inventory. However, that cannot be
used for a manufacturer. We are not told whether this company is a retailer or a manufacturer, so it would not
be a good idea to assume that it is a retailer. We need to work with what we have, and the only thing we have
to use is cost of goods sold. So we will use cost of goods sold in the place of purchases.

$24,500 ÷ $3,500 = 7 times

D.

The accounts payable turnover ratio is total annual credit purchases divided by average accounts payable. We can
calculate the average accounts payable from the information given as the average of the beginning and ending
accounts payable balances ([$3,320 + $3,680] ÷ 2 = $3,500). Purchases for a reseller (retailer) can be approximated
by adjusting cost of goods sold by the amount of change in inventory. However, that cannot be used for a
manufacturer. We are not told whether this company is a retailer or a manufacturer, so it would not be a good idea to
assume that it is a retailer. We need to work with what we have, and the only thing we have to use is cost of goods
sold. So we will use cost of goods sold in the place of purchases.

This is total credit sales divided by ending accounts payable.

Question 53 - CMA 1287 4.1 - Ratios: Liquidity, Leverage, Coverage and Activity

A. Because the income statement records the effects of transactions over a period of time, income statement amounts
cannot be averaged.

B. Comparisons with industry ratios are meaningful, even in the situation in which the ratio compares a balance sheet
amount to an income statement amount.

C. When balance sheet amounts are used together with an income statement amount, the balance sheet
amount used should be an average for the same period as is covered by the income statement item. This is
important because it prevents the ratio from being distorted by an unusually high or low balance sheet
amount at the end of the year.

D. There is no market value for an income statement item and it may be very difficult, if not impossible, to convert
some balance sheet items to a market value.

Question 54 - CMA 1280 4.4 - Ratios: Liquidity, Leverage, Coverage and Activity

A. The writing off of obsolete inventory does not affect the quick ratio because inventory is not included in the
calculation of the quick ratio.

B. The writing off of obsolete inventory does not affect the quick ratio because inventory is not included in the
calculation of the quick ratio.

C. The writing off of obsolete inventory will decrease the current ratio because the balance in the inventory
account is being reduced and there is no change to the current liabilities. This reduction in the numerator of
the current ratio will decrease the current ratio.

D. The writing off of obsolete inventory will decrease working capital because it reduces the amount of the company's
current assets.

(c) HOCK international, page 66


Part 2 : 11/10/17 09:52:04

Question 55 - CMA 1291 1.9 - Ratios: Liquidity, Leverage, Coverage and Activity

A.

The price-earnings ratio is the market price of the firm's stock divided by its basic earnings per share. When the
price-earnings ratio increases, it is because the market price has become relatively higher than the firm's earnings per
share. That may mean that the stock's market price has increased, or it may not mean that at all. It could mean that the
firm's earnings per share has decreased and the stock's market price has decreased also but relatively less than its
EPS has decreased.

Therefore, we cannot conclude anything about the price a firm might receive from issuing new equity by simply looking
at a change in its price-earnings ratio. Since a change in the P-E ratio does not tell us anything about the price a firm
might receive from a new issue of equity, it also does not tell us anything about whether the firm might or might not be
likely to increase its use of debt financing.

B. Operating leverage measures the use of fixed operating costs to generate greater operating profit. A firm with higher
fixed costs will have a higher degree of operating leverage. In order for high operating leverage to be successful, the
company must earn a contribution margin that is high enoiugh to cover the high fixed expenses. Therefore, the higher
a firm's operating leverage is, the more vulnerable it is to a decrease in revenue impacting its profitability negatively. As
the operating leverage increases, bondholders will perceive a greater risk of default and will require a higher rate of
interest to make an investment in the bonds. The increased cost of the bonds will not be likely to cause the firm to
increase its use of debt financing.

C. If the corporate income tax rate increases, the company will use more debt as its source of financing.
Because the interest on the debt is tax deductible, the higher tax rate reduces the effective interest rate for
debt.

D. When there is uncertainty about the future, the company would prefer equity financing because dividend payments
are not required, whereas interest payments are required. Therefore, if the future is poor for the company and they
have issued equity, they will not need to make any dividend payments. However, if they issued debt, they would still be
required to make interest payments, even if the business is not doing well.

Question 56 - CMA 693 2.1 - Ratios: Liquidity, Leverage, Coverage and Activity

A. This answer does not include the receivables in the numerator of the calculation.

B. The acid test (or quick) ratio is calculated as follows: (Cash + Receivables + Trading Securities) / Current
Liabilities. Given the information in this question, we get ($30,000 + $20,000 + $45,000) / $85,000. This is 1.1.

C. This answer incorrectly includes inventory in the calculation of the numerator.

D. This answer includes all current assets in the calculation, so it is the current ratio, not the acid test (quick) ratio.

Question 57 - ICMA 13.P2.010 - Ratios: Liquidity, Leverage, Coverage and Activity

A. The receivables turnover ratio decreased, which indicates that receivables are being collected more slowly
in the current year. If sales increased, that increase could be due to more favorable credit policies which could
mean that more customers with slower payment records received credit.

B. There is no information given that would lead to a conclusion that more stores were open.

(c) HOCK international, page 67


Part 2 : 11/10/17 09:52:04

C. Sales would not increase as a result of more control over inventory levels.

D. There is no information in the question that would indicate any changes in pricing.

Question 58 - CIA 1196 IV.35 - Ratios: Liquidity, Leverage, Coverage and Activity

A.

Before we can solve for fixed assets, we need to solve for accounts receivable and inventory.

To solve for accounts receivable, we need to use the quick ratio we are given, which is 1.2. The quick ratio is
equal to (Cash + Marketable Securities + A/R) ÷ current liabilities. We can calculate that current liabilities equal
$30. This is because total assets are 100 and we know that equity + liabilities is equal to assets. So if liabilities
and equity are 100 and long-term liabilities ($40) plus equity ($30) equal 70, then current liabilities must be $30.
There are no marketable securities, so putting cash and accounts receivable into the formula for the Quick
Ratio, we get [($10 + X) ÷ $30 = 1.2]. Solving for X we get X = $26, and this is the balance of accounts
receivable.

The next balance we need to find is inventory. To determine inventory, we need to use the current ratio
formula of Current Assets ÷ Current Liabilities. We know that this is equal to 1.4. Using the formula we get
[($10 + $26 + X) ÷ 30 = 1.4]. Solving for X, we get X = $6 and this is the inventory balance.

Finally, fixed assets is the balancing amount on the asset side. We have total assets of $100, cash of $10, A/R
of $26, and inventory of $6. $100 − $10 − $26 − $6 = $58, and this is the balance of fixed assets.

There is often more than one way to solve a problem, and on a timed exam, shorter is better. Here is another,
shorter method of calculating the answer, contributed by one of our students:

Current Assets ÷ Current Liabilities = 1.4.

On the liabilities and equity side of the balance sheet, current liabilities is the only figure missing. Therefore,
we can calculate that current liabilities is $100 − $30 − $40, or $30. Now, we can solve for current assets:

Current Assets ÷ 30 = 1.4


Current Assets = 1.4 × $30
Current Assets = $42

Fixed Assets = Total Assets − Current Assets


Fixed Assets = $100 − $42
Fixed Assets = $58

B.

This is not the correct answer. Please see the correct answer for a complete explanation.

We have been unable to determine how to calculate this incorrect answer choice. If you have calculated it, please let
us know how you did it so we can create a full explanation of why this answer choice is incorrect. Please send us an
email at support@hockinternational.com. Include the full Question ID number and the actual incorrect answer choice --
not its letter, because that can change with every study session created. The Question ID number appears in the upper
right corner of the ExamSuccess screen. Thank you in advance for helping us to make your HOCK study materials
better.

C.

This is not the correct answer. Please see the correct answer for a complete explanation.

(c) HOCK international, page 68


Part 2 : 11/10/17 09:52:04

We have been unable to determine how to calculate this incorrect answer choice. If you have calculated it, please let
us know how you did it so we can create a full explanation of why this answer choice is incorrect. Please send us an
email at support@hockinternational.com. Include the full Question ID number and the actual incorrect answer choice --
not its letter, because that can change with every study session created. The Question ID number appears in the upper
right corner of the ExamSuccess screen. Thank you in advance for helping us to make your HOCK study materials
better.

D.

This is not the correct answer. Please see the correct answer for a complete explanation.

We have been unable to determine how to calculate this incorrect answer choice. If you have calculated it, please let
us know how you did it so we can create a full explanation of why this answer choice is incorrect. Please send us an
email at support@hockinternational.com. Include the full Question ID number and the actual incorrect answer choice --
not its letter, because that can change with every study session created. The Question ID number appears in the upper
right corner of the ExamSuccess screen. Thank you in advance for helping us to make your HOCK study materials
better.

Question 59 - ICMA 10.P2.055 - Ratios: Liquidity, Leverage, Coverage and Activity

A. The inventory turnover ratio is annual cost of goods sold divided by average inventory. Average inventory
is ($6,400 + $7,600) ÷ 2, which is $7,000; so the inventory turnover ratio is $24,500 ÷ $7,000, which equals 3.5
times.

B.

The inventory turnover ratio is annual cost of goods sold divided by average inventory. This is credit sales divided by
average inventory.

Remember that the sale price includes profit for the seller. The cost of sales is the cost of the inventory sold, so that is
the amount that must be used when analyzing inventory turnover.

C.

The inventory turnover ratio is annual cost of goods sold divided by average inventory. This is annual cost of goods
sold divided by the ending inventory.

Whenever we relate an income statement amount such as cost of sales to a balance sheet amount such as inventory,
we need to use the average balance of the balance sheet amount, not the beginning or ending balance.

D.

The inventory turnover ratio is annual cost of goods sold divided by average inventory. This is credit sales divided by
year-end inventory.

Remember that the sale price includes profit for the seller. The cost of sales is the cost of the inventory sold, so that is
the amount that must be used when analyzing inventory turnover. Furthermore, whenever we relate an income
statement amount such as cost of sales to a balance sheet amount such as inventory, we need to use the average
balance of the balance sheet amount, not the ending balance.

Question 60 - CMA 695 2.1 - Ratios: Liquidity, Leverage, Coverage and Activity

(c) HOCK international, page 69


Part 2 : 11/10/17 09:52:04

A. The current ratio is calculated as current assets divided by current liabilities. This answer does not include the
prepaid expenses in the current assets.

B. The current ratio is calculated as current assets divided by current liabilities. Current assets include
accounts receivable ($200,000), cash ($100,000), inventory ($400,000) and prepaid expenses ($40,000). Current
liabilities include accounts payable ($80,000), interest payable ($10,000) and notes payable ($50,000). Note that
even though the notes payable are due after the operating cycle is over, the distinction for a current asset or
liability is that it will be converted or settled within 12 months or the operating cycle, whichever is longer.
Thus the 12-month limit governs what is considered a current liability, not the 5-month length of the firm's
operating cycle. The total current assets are $740,000 and current liabilities are $140,000. This gives us a
current ratio of 5.29.

C. The current ratio is calculated as current assets divided by current liabilities. This answer includes the bonds
payable as a current liability.

D. The current ratio is calculated as current assets divided by current liabilities. This answer does not include the
inventory in the current assets.

Question 61 - ICMA 10.P2.039 - Ratios: Liquidity, Leverage, Coverage and Activity

A. A change in earnings cannot cause a change in sales. A change in sales can cause a change in earnings, however.

B. Operating leverage refers to the fact that, for a given level of fixed expenses, a given percentage change in
sales will result in a greater percentage of change in earnings before interest and taxes (EBIT) than the
percentage change in sales. A degree of operating leverage of 3 means that if sales change by 1%, earnings
before interest and taxes will change by 3%.

The formula for Degree of Operating Leverage is: % Change in EBIT ÷ % Change in Revenue, (or alternatively,
Contribution Margin ÷ EBIT). So if the Degree of Operating Leverage is 3, then the percentage of change in the
numerator of that formula must be 3 times as great as the percentage of change in the denominator.
Therefore, a 1% increase in revenue (i.e., sales) causes a 3% increase in EBIT.

C. This is not the meaning of a degree of operating leverage of 3. Operating leverage refers to the fact that, for a given
level of fixed expenses, a given percentage change in sales will result in a greater percentage of change in earnings
before interest and taxes (EBIT) than the percentage change in sales.

D. A change in earnings cannot cause a change in sales. A change in sales can cause a change in earnings, however.

Question 62 - ICMA 10.P2.061 - Ratios: Liquidity, Leverage, Coverage and Activity

A.

Days sales in inventory is calculated by dividing the number of days in the year by the inventory turnover ratio; and
days sales in receivables is calculated by dividing the number of days in the year by the receivables turnover ratio.
However, to answer this question, it is not necessary to go beyond calculating the turnover ratios for the two years for
each. If the turnover ratio increases, the number of days of sales must decrease. And if the turnover ratio decreases,
the number of days of sales must increase. This is because the turnover ratios measure how many times inventory or
accounts receivable "turn over" during a year's time -- for inventory, how many times it is completely sold and replaced
with new inventory, and for accounts receivable, how many times the sales represented are completely paid off and
replaced with new sales. If the number of times the inventory/accounts receivable turn over decrease, then the turnover
is occurring more slowly. As a result, the number of days' sales in the asset balance must have increased. If the
number of times the inventory/accounts receivable turn over increase, then the turnover is occurring more rapidly. As a

(c) HOCK international, page 70


Part 2 : 11/10/17 09:52:04

result, the number of days' sales in the asset balance must have decreased.

Note that we do not have enough information to use the average balances of accounts receivable and inventory for two
years of calculations, so we will have to use the year-end amounts as given in the question for comparing the turnover
ratios for the two years.

The inventory turnover ratio is cost of goods sold divided by average inventory (or here, year-end inventory).

The accounts receivable turnover ratio is sales divided by average accounts receivable (here, year-end accounts
receivable).

B.

Days sales in inventory is calculated by dividing the number of days in the year by the inventory turnover ratio; and
days sales in receivables is calculated by dividing the number of days in the year by the receivables turnover ratio.
However, to answer this question, it is not necessary to go beyond calculating the turnover ratios for the two years for
each. If the turnover ratio increases, the number of days of sales must decrease. And if the turnover ratio decreases,
the number of days of sales must increase. This is because the turnover ratios measure how many times inventory or
accounts receivable "turn over" during a year's time -- for inventory, how many times it is completely sold and replaced
with new inventory, and for accounts receivable, how many times the sales represented are completely paid off and
replaced with new sales. If the number of times the inventory/accounts receivable turn over decrease, then the turnover
is occurring more slowly. As a result, the number of days' sales in the asset balance must have increased. If the
number of times the inventory/accounts receivable turn over increase, then the turnover is occurring more rapidly. As a
result, the number of days' sales in the asset balance must have decreased.

Note that we do not have enough information to use the average balances of accounts receivable and inventory for two
years of calculations, so we will have to use the year-end amounts as given in the question for comparing the turnover
ratios for the two years.

The inventory turnover ratio is cost of goods sold divided by average inventory (or here, year-end inventory).

The accounts receivable turnover ratio is sales divided by average accounts receivable (here, year-end accounts
receivable).

C.

Days sales in inventory is calculated by dividing the number of days in the year by the inventory turnover ratio; and
days sales in receivables is calculated by dividing the number of days in the year by the receivables turnover ratio.
However, to answer this question, it is not necessary to go beyond calculating the turnover ratios for the two years for
each. If the turnover ratio increases, the number of days of sales must decrease. And if the turnover ratio decreases,
the number of days of sales must increase. This is because the turnover ratios measure how many times inventory or
accounts receivable "turn over" during a year's time -- for inventory, how many times it is completely sold and replaced
with new inventory, and for accounts receivable, how many times the sales represented are completely paid off and
replaced with new sales. If the number of times the inventory/accounts receivable turn over decrease, then the turnover
is occurring more slowly. As a result, the number of days' sales in the asset balance must have increased. If the
number of times the inventory/accounts receivable turn over increase, then the turnover is occurring more rapidly. As a
result, the number of days' sales in the asset balance must have decreased.

Note that we do not have enough information to use the average balances of accounts receivable and inventory for two
years of calculations, so we will have to use the year-end amounts as given in the question for comparing the turnover
ratios for the two years.

The inventory turnover ratio is cost of goods sold divided by average inventory (or here, year-end inventory).

The accounts receivable turnover ratio is sales divided by average accounts receivable (here, year-end accounts
receivable).

D.

(c) HOCK international, page 71


Part 2 : 11/10/17 09:52:04

Days sales in inventory is calculated by dividing the number of days in the year by the inventory turnover
ratio; and days sales in receivables is calculated by dividing the number of days in the year by the receivables
turnover ratio. However, to answer this question, it is not necessary to go beyond calculating the turnover
ratios for the two years for each. If the turnover ratio increases, the number of days of sales must decrease.
And if the turnover ratio decreases, the number of days of sales must increase. This is because the turnover
ratios measure how many times inventory or accounts receivable "turn over" during a year's time -- for
inventory, how many times it is completely sold and replaced with new inventory, and for accounts receivable,
how many times the sales represented are completely paid off and replaced with new sales. If the number of
times the inventory/accounts receivable turn over decrease, then the turnover is occurring more slowly. As a
result, the number of days' sales in the asset balance must have increased. If the number of times the
inventory/accounts receivable turn over increase, then the turnover is occurring more rapidly. As a result, the
number of days' sales in the asset balance must have decreased.

Note that we do not have enough information to use the average balances of accounts receivable and
inventory for two years of calculations, so we will have to use the year-end amounts as given in the question
for comparing the turnover ratios for the two years.

The inventory turnover ratio is cost of goods sold divided by average inventory (or here, year-end inventory).
So the inventory turnover ratio for the prior year is 6 ÷ 4, which equals 1.5. The inventory turnover ratio for the
current year is 7 ÷ 5, or 1.4, which is a decrease. Therefore, the number of days of sales in inventory must
have increased.

The accounts receivable turnover ratio is sales divided by average accounts receivable (here, year-end
accounts receivable). The accounts receivable turnover ratio for the prior year is 10 ÷ 3, or 3.33. The accounts
receivable turnover ratio for the current year is 11 ÷ 4, or 2.75. The accounts receivable turnover ratio has
decreased, so the number of days of sales in inventory must have increased.

Question 63 - CIA 1196 IV.34 - Ratios: Liquidity, Leverage, Coverage and Activity

A. This is the amount of working capital (current assets − current liabilities). See the correct answer for a complete
explanation.

B.

We are told that the Quick Ratio is 1.2. This is equal to (Cash + A/R) ÷ Current Liabilities. We can calculate that
current liabilities equal $30. This is because total assets are $100 and we know that equity + liabilities is equal
to assets. So if liabilities and equity are $100 and long-term liabilities ($40) plus equity ($30) equal $70, then
current liabilities must be $30.

The Quick Ratio is (Cash + Marketable Securities + Net Accounts Receivable) ÷ Current Liabilities. We know
what cash is ($10); there are no marketable securities; and the unknown is accounts receivable.

Let X stand for accounts receivable. The formula is:

($10 + X) ÷ $30 = 1.2


$10 + X = $36
X = $26

C. This is the measure of the quick assets of the company. See the correct answer for a complete explanation.

D.

This is not the correct answer. Please see the correct answer for a complete explanation.

(c) HOCK international, page 72


Part 2 : 11/10/17 09:52:04

We have been unable to determine how to calculate this incorrect answer choice. If you have calculated it, please let
us know how you did it so we can create a full explanation of why this answer choice is incorrect. Please send us an
email at support@hockinternational.com. Include the full Question ID number and the actual incorrect answer choice --
not its letter, because that can change with every study session created. The Question ID number appears in the upper
right corner of the ExamSuccess screen. Thank you in advance for helping us to make your HOCK study materials
better.

Question 64 - CMA 1289 P4 Q17 - Ratios: Liquidity, Leverage, Coverage and Activity

A. A stock dividend does not affect total equity. See the correct answer for a complete explanation.

B. A stock dividend does not affect total equity. See the correct answer for a complete explanation.

C. A stock dividend does not affect total equity. See the correct answer for a complete explanation.

D. A stock dividend does not result in a change to total equity. In the stock dividend there is a debit to
Retained Earnings (decreasing total equity) and a credit that is made to Common Stock and Additional Paid in
Capital (increasing total equity). So, a stock dividend does not change total equity.

Question 65 - CMA 1285 4.23 - Ratios: Liquidity, Leverage, Coverage and Activity

A. This transaction would have no impact on the current ratio because all that is happening is the current asset of
accounts receivable is being converted into the current asset of cash.

B. Currently, the current ratio is 0.8. This transaction would increase current assets to $500,000 and current
liabilities to $600,000. After this, the current ratio would be 0.833, an increase.

C. Currently, the current ratio is 0.8. This transaction would increase current liabilities to $600,000 and current assets
would remain unchanged. After this, the current ratio would be 0.667, a decrease, not an increase.

D. Currently, the current ratio is 0.8. This transaction would decrease current assets to $300,000 and current liabilities
to $400,000. After this, the current ratio would be 0.75, a decrease, not an increase.

Question 66 - CMA 696 1.15 - Ratios: Liquidity, Leverage, Coverage and Activity

A. If Spotech is able to turn their inventory over more frequently, they will have a lower average inventory and they will
be able to invest the money that is no longer invested in inventory at 5%. The correct answer is 5% of the amount by
which average inventory will decrease if the company is able to turn its inventory over 10 times per year instead of 8
times. This answer uses an investment rate of 10% instead of 5%.

B.

This is not the correct answer. Please see the correct answer for an explanation.

We have been unable to determine how to calculate this incorrect answer choice. If you have calculated it, please let
us know how you did it so we can create a full explanation of why this answer choice is incorrect. Please send us an
email at support@hockinternational.com. Include the full Question ID number and the actual incorrect answer choice --
not its letter, because that can change with every study session created. The Question ID number appears in the upper
right corner of the ExamSuccess screen. Thank you in advance for helping us to make your HOCK study materials

(c) HOCK international, page 73


Part 2 : 11/10/17 09:52:04

better.

C. If Spotech is able to turn their inventory over more frequently, they will have a lower average inventory and they will
be able to invest the money that is no longer invested in inventory at 5%. This answer is 0.5% of the average amount
that is currently invested in inventory. The correct answer is 5% of the amount by which average inventory will
decrease if the company is able to turn its inventory over 10 times per year instead of 8 times. See the correct answer
for a complete explanation.

D.

If Spotech is able to turn their inventory over more frequently, they will have a lower average inventory and
they will be able to invest the money that is no longer invested in inventory at 5%. In order to answer this
question, we need to calculate what the average inventory is with an inventory turnover of 8 times and what it
would be if the inventory turnover were 10 times instead. Then, we can calculate how much the company
could earn by investing the difference for one year at 5%.

Inventory turnover is calculated as cost of sales divided by average inventory. Therefore, we can calculate
average inventory by dividing cost of sales by the inventory turnover. Currently, average inventory is
$16,562,500 ($132,500,000 ÷ 8). If inventory turnover increases to 10 times, the average inventory will be
$13,250,000 ($132,500,000 ÷ 10). This difference of $3,312,500 will be able to be invested at 5%, earning a total
of $165,625.

Question 67 - CMA 691 2.8 - Ratios: Liquidity, Leverage, Coverage and Activity

A. The total debt-to-equity ratio is calculated as the total liabilities of the company divided by the equity of the company,
consisting of stock and retained earnings in this question. This answer reverses the formula and divides the equity by
the debt.

B. The total debt-to-equity ratio is calculated as the total liabilities of the company divided by the equity of the company,
consisting of stock and retained earnings in this question. This answer does not include the retained earnings of the
company in the denominator.

C. The total debt-to-equity ratio is calculated as the total liabilities of the company divided by the total equity
of the company, consisting of stock and retained earnings in this question. The liabilities of the company were
$790 and the equity was $607 ($226 of outstanding stock and $381 of retained earnings). Dividing $790 by $607
result in a total debt-to-equity ratio of 1.30.

D. The total debt-to-equity ratio is calculated as the total liabilities of the company divided by the equity of the company,
consisting of stock and retained earnings in this question. This answer does not include the stock in the denominator.

Question 68 - CMA 688 4.2 - Ratios: Liquidity, Leverage, Coverage and Activity

A. The debt-to-total asset ratio is total liabilities (current and non-current) divided by total assets. The
budgeted debt for 20X1 is equal to $153,000 ($75,000 + $78,000) and the total budgeted assets are $485,000
($210,000 + $275,000). Thus, the ratio is 0.315.

B. This is total debt for 20X0 divided by total assets for 20X1. Total debt for 20X1 should be used.

C. This is the debt-to-total asset ratio for 20X0.

D. This is total debt for 20X1 divided by total assets for 20X0. Total assets for 20X1 should be used.

(c) HOCK international, page 74


Part 2 : 11/10/17 09:52:04

Question 69 - CMA 690 4.20 - Ratios: Liquidity, Leverage, Coverage and Activity

A. These ratios do not give information about the attitudes of the companies concerning risk.

B. These ratios do not give information about the attitudes of the companies concerning risk.

C. These ratios do provide information about the companies' attitudes towards risk. Because Matson is less
leveraged (it has a lower debt-to-equity ratio) and it has a higher times-interest-earned ratio, we know that
Matson is more conservative. Matson is taking on less debt which decreases the chances of defaulting on its
debt and also has less interest payments given its level of income, thereby also reducing its risk.

D. These ratios do not give information about the attitudes of the companies concerning risk.

Question 70 - ICMA 10.P2.056 - Ratios: Liquidity, Leverage, Coverage and Activity

A. The inventory turnover ratio is Cost of Goods Sold ÷ Average Inventory. Average inventory is the average of
the beginning and ending inventories, which is ($125,000 + $138,000) ÷ 2, or $131,500. So the inventory
turnover ratio is $527,000 ÷ $131,500, which equals 4.01.

B. The inventory turnover ratio is Cost of Goods Sold ÷ Average Inventory. This is Sales ÷ Average Inventory.

C. The inventory turnover ratio is Cost of Goods Sold ÷ Average Inventory. This is Cost of Goods Sold ÷ Ending
Inventory.

D. The inventory turnover ratio is Cost of Goods Sold ÷ Average Inventory. This is Sales ÷ Ending Inventory.

Question 71 - CIA 596 IV.53 - Ratios: Liquidity, Leverage, Coverage and Activity

A. This is 58 days to convert the raw materials into finished goods minus 32 days to collect the receivables minus 15
days to pay for the raw materials. The number of days required to collect the receivables increases the total cash
conversion cycle rather than decreasing it.

B. This is 58 days to convert the raw materials into finished goods plus 32 days to collect the receivables. The number
of days the company takes to pay for the raw materials is a part of the cash conversion cycle, too, and that has not
been included.

C. This is 58 days to convert the raw materials into finished goods minus 32 days to collect the receivables plus 15
days to pay for the raw materials. The number of days required to collect the receivables increases the total cash
conversion cycle rather than decreasing it; and the number of days to pay for the raw materials decreases the total
cash conversion cycle rather than increasing it.

D. The cash conversion cycle is equal to the days in inventory plus the days to collect receivables minus the
days to pay payables. For a manufacturing company such as this one, the number of days in raw materials
inventory are included as well. Therefore, the cash cycle is 75 days (58 + 32 − 15).

Question 72 - CMA 693 2.4 - Ratios: Liquidity, Leverage, Coverage and Activity

(c) HOCK international, page 75


Part 2 : 11/10/17 09:52:04

A. The inventory turnover is calculated as annual cost of sales divided by the average annual inventory. This answer
was calculated using the 20X3 year-end inventory instead of the average inventory during the year.

B. The inventory turnover is calculated as annual cost of sales divided by the average annual inventory. This answer
was calculated using the 20X4 year-end inventory instead of the average inventory during the year.

C. The inventory turnover is calculated as annual cost of sales divided by the average annual inventory. The
problem gives the annual cost of sales as $220,000. The average inventory is $55,000 (the average of $60,000
and $50,000, the two year-end inventory balances). The inventory turnover is calculated as $220,000 divided
by $55,000, or 4 times. This means that Lisa sells their inventory four times a year.

D. The inventory turnover is calculated as annual cost of sales divided by the average annual inventory. This answer
was calculated using the 20X4 sales of $300,000 instead of the 20X4 cost of sales of 220,000, and using 20X4
year-end inventory instead of the average inventory during the year.

Question 73 - ICMA 10.P2.030 - Ratios: Liquidity, Leverage, Coverage and Activity

A. The current ratio is current assets divided by current liabilities. This answer results from including deferred taxes as
a current liability. Deferred taxes cannot be included in either current assets or current liabilities in this question.
Deferred taxes could be an asset, or they could be a liability. If they are an asset, they could be either a current asset
or a long-term asset. If they are a liability, they could be either a current liability or a long-term liability. There is not
enough information given in the question to enable us to know what they are, so there is nothing we can do but ignore
them.

B. The current ratio is current assets divided by current liabilities. This answer results from omitting inventory in
calculating current assets and including only accounts payable as current liabilities.

C.

The current ratio is current assets divided by current liabilities. Trading securities are usually classified as
current assets, although they may be classified as non-current assets if management does not consider them
available to be used for current operations. Here, we will assume they are a current asset.

Current assets = Cash $62,000 + A/R $47,000 + Trading Securities $35,000 + Inventory $138,000 = $282,000.
Current liabilities = Accounts Payable $84,000 + Accrued Liabilities $11,000 = $95,000. The current ratio is
$282,000 ÷ $95,000 = 2.97.

Deferred taxes are not included in either current assets or current liabilities in this question. Deferred taxes
could be an asset, or they could be a liability. If they are an asset, they could be either a current asset or a
long-term asset. If they are a liability, they could be either a current liability or a long-term liability. There is not
enough information given in the question to enable us to know what they are, so there is nothing we can do
but ignore them.

D.

The current ratio is current assets divided by current liabilities. This answer results from including deferred taxes and
long-term bonds payable as current liabilities.

Deferred taxes cannot be included in either current assets or current liabilities in this question. Deferred taxes could be
an asset, or they could be a liability. If they are an asset, they could be either a current asset or a long-term asset. If
they are a liability, they could be either a current liability or a long-term liability. There is not enough information given in
the question to enable us to know what they are, so there is nothing we can do but ignore them.

Long-term bonds payable are long-term liabilities. Long-term liabilities are not included in the calculation of the current
ratio.

(c) HOCK international, page 76


Part 2 : 11/10/17 09:52:04

Question 74 - ICMA 10.P2.032 - Ratios: Liquidity, Leverage, Coverage and Activity

A. Cash is a current asset, and if it decreases, both the current ratio and the quick ratio would decrease, because cash
is included in the numerator of both ratios. Since it is included in both ratios, cash cannot account for the disparity
(difference) between the current and quick ratios.

B. Accounts receivable are current assets, and if they decrease, both the current ratio and the quick ratio would
decrease, because accounts receivable are included in the numerator of both ratios. Since they are included in both
ratios, accounts receivable cannot account for the disparity (difference) between the current and quick ratios.

C. Current portion of long-term debt is a current liability, and if it increases, both the current ratio and the quick ratio
would decrease. Since the current portion of long-term debt is included in the denominator of both ratios, it cannot
account for the disparity (difference) between the current and quick ratios.

D. Inventory is included in the numerator of the current ratio but it is not included in the numerator of the
quick ratio. Thus, inventory accounts for the disparity (difference) between the current and quick ratios.

Question 75 - CIA 1193 IV.48 - Ratios: Liquidity, Leverage, Coverage and Activity

A. Because of the retirement of the bonds is done by spending cash, the total assets of the company will decrease.
This decrease in total assets will increase the asset turnover ratio.

B. Financial leverage relates to the amount of debt that a company uses for its financing. As the amount of
debt decreases, the leverage of the company will decrease. The fact that the bonds will be retired at 103.5% of
the face value is not relevant to this question.

C. Because the amount of debt will be decreased and their will be no change in equity, the debt-equity ratio will
decrease.

D. Because the bonds will be retired there will be less interest expense which will increase net income. This increase in
net income will cause the return on owners equity to increase.

Question 76 - CMA 1293 2.13 - Ratios: Liquidity, Leverage, Coverage and Activity

A. Inventory turnover is calculated as cost of sales divided by the average inventory for the period. If sales were used
instead of cost of sales, this ratio would change simply as a result of the company changing the price that they sell the
product for. Therefore, sales is not a good base to use.

B. Inventory turnover is calculated as cost of sales divided by the average inventory for the period. If sales were used
instead of cost of sales, this ratio would change simply as a result of the company changing the price that they sell the
product for. Therefore, sales is not a good base to use.

C. Inventory turnover is calculated as cost of sales divided by the average inventory for the period. If sales
were used instead of cost of sales, this ratio would change simply as a result of the company changing the
price that they sell the product for. Therefore, using cost of sales instead insulates this ratio from a change in
the selling price of the product.

D. Inventory turnover is calculated as cost of sales divided by the average inventory for the period. If sales were used
instead of cost of sales, this ratio would change simply as a result of the company changing the price that they sell the

(c) HOCK international, page 77


Part 2 : 11/10/17 09:52:04

product for. Therefore, sales is not a good base to use.

Question 77 - CMA 688 4.3 - Ratios: Liquidity, Leverage, Coverage and Activity

A. This is sales divided by beginning accounts receivable ($70,000 as of May 31, 20X0). Average balances of balance
sheet items are used instead of beginning or ending balances whenever a ratio calculation is relating an income
statement amount to a balance sheet amount. This average balance amount should be the average balance of the
balance sheet item during the same period of time as the income statement amount covers. This makes the
relationship of the two amounts meaningful. The average balance is usually calculated as the average of the beginning
and ending balances of the period. Therefore, the average accounts receivable balance for 20X1 is the average of the
ending 20X0 and the ending 20X1 accounts receivable balances.

B. This is cost of goods sold divided by average accounts receivable. Accounts receivable turnover is net credit sales
divided by average accounts receivable.

C. Accounts receivable turnover is net credit sales divided by average accounts receivable. For McKeon,
budgeted credit sales for 20X1 are $350,000 and the average accounts receivable is $85,000 ($70,000 actual at
the end of 20X0 and $100,000 projected at the end of 20X1). This gives us $350,000 ÷ $85,000 = 4.118.

D.

This answer uses the expected 20X1 ending accounts receivable in the calculation instead of the average accounts
receivable balance for 20X1.

Average balances of balance sheet items are used instead of beginning or ending balances whenever a ratio
calculation is relating an income statement amount to a balance sheet amount. This average balance amount should
be the average balance of the balance sheet item during the same period of time as the income statement amount
covers. This makes the relationship of the two amounts meaningful.

The average balance is usually calculated as the average of the beginning and ending balances of the period.
Therefore, the average accounts receivable balance for 20X1 is the average of the ending 20X0 and the ending 20X1
accounts receivable balances.

Question 78 - CMA 688 4.11 - Ratios: Liquidity, Leverage, Coverage and Activity

A. The inventory turnover does not measure the ability of a company to pay its debt.

B. Times interest earned, also called the interest coverage ratio, is calculated as earnings before interest and
taxes divided by the interest expense the company incurs. This does measure the ability of the company to
pay its debt.

C. The return on assets does not measure the ability of a company to pay its debt.

D. The length of the operating cycle does not measure the ability of a company to pay its debt.

Question 79 - CMA 688 4.12 - Ratios: Liquidity, Leverage, Coverage and Activity

A. This is Ending Inventory × 8. The inventory turnover ratio (8.0) needs to be multiplied by average inventory, which is
(Beginning Inventory + Ending Inventory) ÷ 2.

(c) HOCK international, page 78


Part 2 : 11/10/17 09:52:04

B.

This is (Beginning Inventory − Ending Inventory) × 8. The inventory turnover ratio (8.0) needs to be multiplied by
average inventory, which is (Beginning Inventory + Ending Inventory) ÷ 2.

C.

Cost of sales divided by average inventory equals inventory turnover. The problem tells us what the inventory
turnover is, and if we can find what average inventory is, we can calculate the cost of sales.

We have the beginning inventory balance. To find what average inventory is, we need to know the ending
inventory balance so we can calculate their average. If current liabilities are $600,000 and the current ratio is
3.5, that means that current assets are $2,100,000 ($600,000 × 3.5). Given an acid test ratio of 3.0 and current
liabilities of $600,000, we know that the numerator of the quick ratio is $1,800,000 ($600,000 × 3.0).

The primary difference between the numerator of the current ratio and the numerator of the quick ratio is
inventory. Thus, ending inventory is $300,000 ($2,100,000 − $1,800,000). Since ending inventory was $300,000
and beginning inventory was $500,000, we can calculate that we have an average inventory of $400,000.

Now we have both inventory turnover and average inventory. Inventory turnover is calculated as cost of sales
divided by average inventory. Using the inventory turnover of 8, we know that the cost of sales was 8 times
the average inventory of $400,000, or $3,200,000.

D.

This is (Beginning Inventory + Ending Inventory) × 8. The inventory turnover ratio (8.0) needs to be multiplied by
average inventory, which is (Beginning Inventory + Ending Inventory) ÷ 2.

Question 80 - CMA 1280 4.1 - Ratios: Liquidity, Leverage, Coverage and Activity

A. The quick ratio would be affected by this transaction as both the numerator and denominator of the quick ratio would
be decreased by the amount of the liability settled.

B. Because both of these ratios were greater than 1.0 prior to this transaction, both of these ratios will increase as a
result of the settlement of the liability.

C. The current ratio would be affected by this transaction as both the numerator and denominator of the current ratio
would be decreased by the amount of the liability settled.

D. Because Depoole has a quick and current ratio of greater than one (because total quick assets exceeded
total current liabilities), the payment of a payable would increase both ratios. This is because the payment of a
payable will reduce both the current assets and the current liabilities by the same amount. Let us assume that
Depoole had current and quick assets of $150 and current liabilities of $100. Given that the current and quick
assets were both $150, both ratios are 1.50. Now let assume that they paid a $20 liability. This reduced the
quick and current assets to $130 and reduced the current liabilities to $80. Now, both ratios are 1.625, which is
higher than they had been.

Question 81 - ICMA 10.P2.027 - Ratios: Liquidity, Leverage, Coverage and Activity

A. This answer does not include marketable securities as part of current assets.

(c) HOCK international, page 79


Part 2 : 11/10/17 09:52:04

B. This answer does not include supplies as part of current assets and it does not include accounts payable as part of
current liabilities.

C. This answer does not include accruals as part of current liabilities.

D. Net working capital is Current Assets – Current Liabilities. Current assets in this case are made up of
marketable securities (10,000) + accounts receivable ($60,000) + inventory ($25,000) + supplies ($5,000) for a
total of $100,000. Current liabilities are accounts payable ($40,000) + short term debt ($10,000) + accruals
($5,000) for a total of $55,000. Net working capital = $45,000.

Question 82 - CMA 690 4.13 - Ratios: Liquidity, Leverage, Coverage and Activity

A. The operating cycle is the length of time it takes to convert an investment of cash in inventory back into cash
(through collections of sales). It is calculated as the days sales in inventory + days sales in receivables. This
information will not allow us to calculate the operating cycle.

B. The operating cycle is the length of time it takes to convert an investment of cash in inventory back into cash
(through collections of sales). It is calculated as the days sales in inventory + days sales in receivables. In addition to
the average merchandise inventory, we also need to know the daily cost of goods sold in order to calculate the days
sales in inventory.

C. The operating cycle is the length of time it takes to convert an investment of cash in inventory back into cash
(through collections of sales). It is calculated as the days sales in inventory + days sales in receivables. This
information will not allow us to calculate the operating cycle.

D. The operating cycle is the length of time it takes to convert an investment of cash in inventory back into
cash (through collections of sales). It is calculated as the days sales in inventory + days sales in receivables.
Both of these amounts can be calculated from the information provided in this choice. The days sales in
receivables is 365 divided by the accounts receivable turnover, and the days sales in inventory is 365 divided
by the inventory turnover.

Question 83 - HOCK MP1 E21 - Ratios: Liquidity, Leverage, Coverage and Activity

A. An answer of 9 days results from dividing total credit sales of $1,500,000 by the accounts receivable balance of
$165,000. However, that is not the way to calculate the number of days of sales in receivables.

B. An answer of 20 days results from using the total sales as the total credit sales in calculating the number of days of
sales in receivables. The question says that only half of the sales are made on credit, however.

C.

The first step is to take half of the total sales for the year, since only half of the $3,000,000 sales are made on
credit. The next step is to find the average daily sales made on credit. To do this, we divide $1,500,000 (half of
$3,000,000) by 365 days, which equals $4,109.59. Next, divide the accounts receivable balance of $165,000 by
the average daily credit sales of $4,109.59, which equals 40.15, or 40 days.

The average collection period can also be calculated by dividing 365 by the receivables turnover ratio. The
receivables turnover ratio is $1,500,000 ÷ $165,000, which is 9.0909. Dividing 365 by 9.0909, we get 40.15, or 40
days.

NOTE: This question does not give the average balance of accounts receivable, nor does it give information
that would enable us to calculate the average balance of accounts receivable. We have used the ending

(c) HOCK international, page 80


Part 2 : 11/10/17 09:52:04

accounts receivable balance, since that is all we have. However, the average balance of accounts receivable
should be used whenever possible.

D. An answer of 18 days results from dividing total sales of $3,000,000 by the accounts receivable balance of
$165,000. However, that is not the way to calculate the number of days of sales in receivables.

Question 84 - CMA 688 4.4 - Ratios: Liquidity, Leverage, Coverage and Activity

A. In order to calculate the days sales in inventory for 20X1, we first need to calculate how many times the inventory
turns over during the year. This is COGS divided by average inventory. This answer uses the year-end inventory
instead of the average inventory in the calculation of the number of days of sales in inventory.

B. In order to calculate the days sales in inventory for 20X1, we first need to calculate how many times the inventory
turns over during the year. This is COGS divided by average inventory. This answer uses the beginning 20X1 inventory
(the 20X0 year-end inventory) instead of the average inventory for 20X1 in the calculation of the number of days of
sales in inventory.

C. In order to calculate the days sales in inventory for 20X1, we first need to calculate how many times the
inventory turns over during the year. This is COGS divided by average inventory. COGS is budgeted at
$160,000 and average inventory is $75,000 (the average of $70,000 actual at year-end 20X0 and $80,000
planned for year-end 20X1). This gives us an inventory turnover ratio of $160,000 ÷ $75,000 = 2.13 times per
year. If the inventory turns over 2.13 times during the year, then the days of sales in inventory equals 171 days
(365 ÷ 2.13).

D. This answer results from using the sales revenue of $350,000 instead of cost of goods sold of $160,000 in the
calculation.

Question 85 - CMA 1280 4.3 - Ratios: Liquidity, Leverage, Coverage and Activity

A. The collection of a receivable will not change the current ratio. Cash will increase by the same amount that
receivables decrease, so current assets will be unchanged. See the correct answer for a complete explanation.

B. The collection of a receivable will not change the current or quick ratios. Cash will increase by the same amount that
receivables decrease, so both current assets and quick assets will be unchanged. See the correct answer for a
complete explanation.

C. When a receivable is collected, there is no change in the quick or the current ratios. This is because this
transaction does not change the level of current assets or current liabilities. When a receivable is collected,
the company is simply exchanging one current and quick asset (receivables) for another current and quick
asset (cash) of the same amount. Therefore, the current and quick ratios do not change.

D. The collection of a receivable will not change the quick ratio. Cash will increase by the same amount that
receivables decrease, so quick assets will be unchanged. See the correct answer for a complete explanation.

Question 86 - CMA 690 1.9 - Ratios: Liquidity, Leverage, Coverage and Activity

A. Financial leverage is total assets divided by total equity. Issuing common stock and using the proceeds to reetire
preferred stock would not cause financial leverage to increase.

(c) HOCK international, page 81


Part 2 : 11/10/17 09:52:04

B.

Using more bonds in the future would increase the financial leverage. Financial leverage is total assets
divided by total equity. Using more bonds would increase total liabilities in relation to total equity and assets
would increase from the proceeds of the bonds. The result would be higher financial leverage.

C. Financial leverage is total assets divided by total equity. If the dividend payout ratio is decreased, retained earnings
will increase and equity will increase. All other things being equal, the financial leverage of the company will decrease.

D. The use of equity as the source of capital leads to a decreased financial leverage. Financial leverage is total assets
divided by total equity. If total equity increases, all other things being equal, financial leverage will decrease.

Question 87 - ICMA 10.P2.036 - Ratios: Liquidity, Leverage, Coverage and Activity

A. The current ratio is current assets divided by current liabilities. If a company's current ratio is 2.1, that means its
current assets are 2.1 times as great as its current liabilities. Paying off a portion of its accounts payable with cash
would decrease current assets and current liabilities by the same amount. The current ratio would not remain
unchanged as a result.

B. The current ratio is current assets divided by current liabilities. If a company's current ratio is 2.1, that means its
current assets are 2.1 times as great as its current liabilities. Paying off a portion of its accounts payable with cash
would decrease current assets and current liabilities by the same amount. The current ratio would not decrease as a
result.

C.

The current ratio is current assets divided by current liabilities. If a company's current ratio is 2.1, that means
its current assets are 2.1 times as great as its current liabilities. Paying off a portion of its accounts payable
with cash would decrease current assets and current liabilities by the same amount. Because current assets
are greater than current liabilities, the same amount of decrease in both will be a smaller percentage of
decrease in current assets than it will current liabilities. As a result the ratio will increase.

As an example, suppose the company has $210,000 in current assets and $100,000 in current liabilities, a 2.1
current ratio. It uses cash to pay off $20,000 in accounts payable, so current assets decrease to $190,000 and
current liabilities decrease to $80,000. The current ratio increases to 2.375.

D. The current ratio is current assets divided by current liabilities. If a company's current ratio is 2.1, that means its
current assets are 2.1 times as great as its current liabilities. Paying off a portion of its accounts payable with cash
would decrease current assets and current liabilities by the same amount. The current ratio would not move closer to
the quick ratio as a result.

Question 88 - CMA 695 1.1 - Ratios: Liquidity, Leverage, Coverage and Activity

A. Operating leverage is a measure of the degree to which fixed costs are used in the production process. A
company with a higher percentage of fixed costs (higher operating leverage) has greater risk than one in the
same industry that relies more heavily on variable costs. The DOL equals the percentage change in net
operating income divided by the percentage change in sales. Thus, profits become more sensitive to changes
in sales volume as the DOL increases.

B. A firm with higher operating leverage has higher fixed costs and lower variable costs.

C. A firm with higher leverage will be relatively more profitable than a firm with lower leverage when sales are high. The

(c) HOCK international, page 82


Part 2 : 11/10/17 09:52:04

opposite is true when sales are low. Therefore, a firm with higher operating leverage may or may not be more profitable
than one with lower operating leverage.

D. A firm with higher operating leverage is more risky than a firm with lower operating leverage because its contribution
margin must be high enough to cover its fixed costs.

Question 89 - ICMA 10.P2.066 - Ratios: Liquidity, Leverage, Coverage and Activity

A. The total asset turnover ratio is sales divided by average total assets. This is sales divided by total assets at the
beginning of the year. Average total assets is the average of the beginning and ending balances.

B. The total asset turnover ratio is sales divided by average total assets. This is sales divided by average assets
excluding cash. Cash is an asset and should be included.

C. The total asset turnover ratio is sales divided by average total assets. Total assets at January 1 were
$608,000; at December 31, they were $706,000. Average total assets equaled ($608,000 + $706,000) ÷ 2, or
$657,000. Sales is given as $900,000. Thus, the total asset turnover is $900,000 ÷ $657,000, which equals 1.37.

D. The total asset turnover ratio is sales divided by average total assets. This is sales divided by year-end total assets.
Average total assets is the average of the beginning and ending balances.

Question 90 - ICMA 10.P2.034 - Ratios: Liquidity, Leverage, Coverage and Activity

A. The acid test or quick ratio is Cash + Marketable Securities + Accounts Receivable divided by Current Liabilities.
Six-month treasury bills are short-term marketable securities and so would be included when calculating the acid test
ratio.

B. The acid test or quick ratio is Cash + Marketable Securities + Accounts Receivable divided by Current Liabilities.
60-day certificates of deposit may be cash equivalents if under $100,000 or marketable securities if $100,000 and over.
Either way, they are included when calculating the acid test ratio.

C. The acid test or quick ratio is Cash + Marketable Securities + Accounts Receivable divided by Current
Liabilities. Prepaid insurance and other prepaids are not included in the calculation of the acid test ratio. Even
though prepaid insurance is presented on the balance sheet as a current asset, it is not an asset that can be
liquidated for cash (unless the insurance policy is cancelled, in which case some refund might be received).
But it is not an easily liquidated current asset that can be routinely used to pay current liabilities.

D. The acid test or quick ratio is Cash + Marketable Securities + Accounts Receivable divided by Current Liabilities.
Accounts receivable are included when calculating the acid test ratio.

Question 91 - ICMA 10.P2.024 - Ratios: Liquidity, Leverage, Coverage and Activity

A. The quick ratio, or acid test ratio, is (Cash + Marketable Securities + Accounts Receivable) ÷ Total Current
Liabilities. This answer omits Accounts Receivable from the numerator.

B. The quick ratio, or acid test ratio, is (Cash + Marketable Securities + Accounts Receivable) ÷ Total Current
Liabilities. This answer includes the 7% bonds payable in current liabilities. Bonds payable are a non-current liability.

C. The quick ratio, or acid test ratio, is (Cash + Marketable Securities + Accounts Receivable) ÷ Total Current

(c) HOCK international, page 83


Part 2 : 11/10/17 09:52:04

Liabilities. Using December 31 amounts, this is ($62,000 + $35,000 + $47,000) ÷ ($84,000 + $11,000) = 1.516 or
rounded, 1.52.

D. The quick ratio, or acid test ratio, is (Cash + Marketable Securities + Accounts Receivable) ÷ Total Current
Liabilities. This answer omits Marketable Securities from the numerator.

Question 92 - CIA 1190 IV.55 - Ratios: Liquidity, Leverage, Coverage and Activity

A. The debt-to-assets ratio is calculated as the total debt divided by the total assets. If the assets are purchased by
using borrowed funds, the ratio will change. See the correct answer for a more detailed explanation.

B. The debt-to-assets ratio is calculated as the total debt divided by the total assets. Let us assume that
currently the company has $100 of debt and $200 of assets. This gives us the 0.50 ratio that currently exists. If
the assets are purchased, both the debt and the assets will increase by the amount of the purchase (let us
assume $50). This will make the ratio $150 / $250, or 0.60 – an increase. If the assets are acquired under an
operating lease there will be no debt recorded and there will be no asset recorded, so the ratio will remain
unchanged.

C. The debt-to-assets ratio is calculated as the total debt divided by the total assets. If the asset is purchased through
the use of borrowed funds, the ratio will change. See the correct answer for a more detailed explanation.

D. The debt-to-assets ratio is calculated as the total debt divided by the total assets. If the asset is leased under an
operating lease, the ratio will remain unchanged as there will be no increases in the level of debt or the level of assets.

Question 93 - ICMA 10.P2.022 - Ratios: Liquidity, Leverage, Coverage and Activity

A. The current ratio is total current assets divided by total current liabilities. When merchandise is purchased on credit,
both inventory (a current asset) and accounts payable (a current liability) will increase. Since the beginning values of
total current assets and total current liabilities are different, the same amount of increase to both will change the ratio
between them.

B. When merchandise is purchased on credit, inventory increases. Inventory is a current asset, so total current assets
increase when merchandise is purchased on credit.

C. When merchandise is purchased on credit, accounts payable increases. Accounts payable is a current liability, so
total current liabilities increase when merchandise is purchased on credit.

D. Net working capital is total current assets minus total current liabilities. When merchandise is purchased
on credit, inventory and accounts payable increase by the same amount. The difference between the two will
be unchanged by the equal increases.

Question 94 - HOCK MP1 E2 - Ratios: Liquidity, Leverage, Coverage and Activity

A. The debt-equity ratio measures how much long-term debt a company has compared to its total equity. A
ratio in excess of 1:1 indicates more reliance on long-term debt financing than on equity financing. Issuing
rights to purchase new common stock would result in increased equity because some of the rights would be
exercised to purchase the newly-issued stock. That would increase equity, which could decrease the
debt-equity ratio.

(c) HOCK international, page 84


Part 2 : 11/10/17 09:52:04

B. Issuing new bonds would increase long-term debt, which would increase the debt-equity ratio. This is the opposite of
management's goal, because the debt-equity ratio now is 1.2:1, and management wants to decrease it to 1:1.

C. Payment of a stock dividend would not change the debt-equity ratio. When a stock dividend is declared, the
corporation does not pay out assets or incur a liability. It issues additional shares of stock to each stockholder and
nothing more. Total equity remains the same.

D. All dividends, except for stock dividends, reduce the total stockholders' equity in the corporation, because the equity
is reduced either through an immediate or promised future distribution of assets. Therefore, payment of a dividend on
its common stock would increase the debt-equity ratio, since it would decrease equity. This is the opposite of
management's goal, because the debt-equity ratio now is 1.2:1, and management wants to decrease it to 1:1.

Question 95 - CIA 594 IV.52 - Ratios: Liquidity, Leverage, Coverage and Activity

A. The degree of total leverage is the multiple of the degree of operating leverage and the degree of financial leverage.
Other things being equal, DOL is higher if the degree of total leverage is higher.

B.

The degree of operating leverage (DOL) is a measure of the change in earnings before interest and taxes
associated with a given change in sales volume. For a particular level of output, Degree of Operating Leverage
is calculated as follows:

% Change in EBIT
% Change in Revenue

C. The degree of total leverage is the multiple of the degree of operating leverage and the degree of financial leverage.
Other things being equal, DOL is higher if the degree of total leverage is higher.

D. The degree of financial leverage is a measure of the change in earnings available to common stockholders
associated with a given change in operating earnings.

Question 96 - CMA 695 2.3 - Ratios: Liquidity, Leverage, Coverage and Activity

A.

The current ratio is calculated as current assets divided by current liabilities. Current assets include accounts
receivable ($200,000), cash ($100,000), inventory ($400,000) and prepaid expenses ($40,000), for a total of
$740,000. Current liabilities include accounts payable ($80,000), interest payable ($10,000) and notes payable
($50,000), for a total of $140,000. Note that even though the notes payable are due after the operating cycle is
over, the distinction for a current asset or liability is that it will be converted or settled within 12 months or the
operating cycle, whichever is longer. Thus the 12-month limit governs what is considered a current liability,
not the 5-month length of the firm's operating cycle. $740,000 divided by $140,000 gives us a current ratio of
5.29 before the payment is made.

If the payment of the 50% of the payables is made, $40,000 will be paid. The total current assets will decrease
by $40,000 to $700,000 and the current liabilities will decrease by the same amount to $100,000. This gives a
new current ratio of 7, an increase in the current ratio.

The quick ratio is calculated as quick assets (cash + marketable securities + accounts receivable) divided by
current liabilities. Quick assets include accounts receivable ($200,000) and cash ($100,000), for a total of
$300,000. Current liabilities as calculated above are $140,000. This gives us a quick ratio of 2.14 before the

(c) HOCK international, page 85


Part 2 : 11/10/17 09:52:04

payment of the payables. If the payment of $40,000 of the payables is made, the total quick assets will
decrease by

$40,000 to $260,000 and the current liabilities will decrease by the same amount to $100,000. This gives a new
quick ratio of 2.6, an increase in the quick ratio.

B. The current ratio is current assets divided by current liabilities. The quick ratio is "quick" assets divided by current
liabilities. In this problem, both ratios are greater than 1 (i.e., their numerators are greater than their denominators). If
cash is used to pay a portion of the accounts payable, the numerator and denominator of each ratio will decrease by
the same amounts in dollars. As a result, the current ratio and the quick ratio will move in the same direction.

C. The current ratio is current assets divided by current liabilities. The quick ratio is "quick" assets divided by current
liabilities. In this problem, both ratios are greater than 1 (i.e., their numerators are greater than their denominators). If
cash is used to pay a portion of the accounts payable, the numerator and denominator of each ratio will decrease by
the same amounts in dollars. As a result, the current ratio and the quick ratio will move in the same direction.

D. The current ratio is current assets divided by current liabilities. The quick ratio is "quick" assets divided by current
liabilities. In this problem, both ratios are greater than 1 (i.e., their numerators are greater than their denominators). If
cash is used to pay a portion of the accounts payable, the numerator and denominator of each ratio will decrease by
the same amounts in dollars. But because the numerators are greater than the denominators, the denominators will
decrease proportionately more than will the numerators. This will not cause both ratios to decrease.

Question 97 - CMA 1293 2.16 - Ratios: Liquidity, Leverage, Coverage and Activity

A. Asset turnover is calculated as sales divided by total assets and is a measure of the company's ability to
generate earnings from all of its resources.

B. Though sales to working capital does measure the effectiveness of the use of working capital, it does not measure
the effectiveness of the use of all of the assets because working capital includes only current assets.

C. Days' sales in inventory measures how quickly the company is able to sell its inventory.

D. Days' sales in receivables measures how effectively the company is collecting its receivables.

Question 98 - CMA 693 2.3 - Ratios: Liquidity, Leverage, Coverage and Activity

A.

This is not the correct answer. Please see the correct answer for an explanation.

We have been unable to determine how to calculate this incorrect answer choice. If you have calculated it, please let
us know how you did it so we can create a full explanation of why this answer choice is incorrect. Please send us an
email at support@hockinternational.com. Include the full Question ID number and the actual incorrect answer choice --
not its letter, because that can change with every study session created. The Question ID number appears in the upper
right corner of the ExamSuccess screen. Thank you in advance for helping us to make your HOCK study materials
better.

B. The average collection period is calculated as average accounts receivable divided by the average daily sales. This
answer was calculated using 20X3 year-end accounts receivable instead of the average accounts receivable during
20X4.

C. The average collection period is calculated as average accounts receivable divided by the average daily sales. This

(c) HOCK international, page 86


Part 2 : 11/10/17 09:52:04

answer was calculated using 20X4 year-end accounts receivable instead of the average accounts receivable during
20X4.

D. The average collection period is calculated as average accounts receivable divided by the average daily
sales. Average receivables were $37,500 ($45,000 + $30,000 divided by 2) and the average daily sales were
$833.33 ($300,000 divided by 360). Dividing $37,500 by $833.33 gives us 45 days of sales in receivables. This
means that it takes an average of 45 days to collect receivables.

Question 99 - ICMA 10.P2.023 - Ratios: Liquidity, Leverage, Coverage and Activity

A. The current ratio is total current assets divided by total current liabilities. When a payment of $100,000 cash is used
to pay $100,000 of accounts payable, both cash (a current asset) and accounts payable (a current liability) will
decrease by $100,000. Since the beginning values of total current assets and total current liabilities are different, the
same amount of decrease to both will change the ratio between them.

B. The quick ratio is the total of cash, marketable securities, and net accounts receivable divided by total current
liabilities. If a payment of $100,000 cash is used to purchase inventory, the numerator of the ratio will change while the
denominator remains the same. That will change the quick ratio.

C. The quick ratio is the total of cash, marketable securities, and net accounts receivable divided by total
current liabilities. If a payment of $100,000 cash is used to purchase inventory, the numerator of the ratio will
decrease while the denominator remains the same. As a result, the quick ratio will decrease.

D. The current ratio is total current assets divided by total current liabilities. Currently, the current ratio is $2,600,000 ÷
$1,300,000, which is 2.0. When a payment of $100,000 cash is used to pay $100,000 of accounts payable, both cash
(a current asset) and accounts payable (a current liability) will decrease by $100,000. Current assets will become
$2,500,000 and current liabilities will become $1,200,000. The current ratio will become $2,500,000 ÷ $1,200,000,
which is 2.083. Therefore, the current ratio will increase, not decrease.

Question 100 - CMA 1289 P4 Q14 - Ratios: Liquidity, Leverage, Coverage and Activity

A.

The current ratio is calculated as current assets divided by current liabilities. Prior to the dividend events, the
current ratio of the company was 3.04 ($5,431,000 ÷ $1,789,000). After the declaration of the dividend, the
current liabilities increased by $750,000 and this would decrease the current ratio to 2.14 ($5,431,000 ÷
$2,539,000).

When the dividend is paid, both current assets and current liabilities will decrease by $750,000, with current
liabilities returning to its pre-dividend level of $1,789,000. Now, the current ratio is 2.62 ($4,681,000 ÷
$1,789,000). Since the current ratio was 2.14 before the dividend payment, the dividend payment increased the
current ratio.

B. The current ratio is changed by both of these events. See the correct answer for a complete explanation.

C. The payment of the dividend increased the current ratio. See the correct answer for a complete explanation.

D. The declaration of the dividend decreased the current ratio, and the payment of the dividend increased the current
ratio. See the correct answer for a complete explanation.

(c) HOCK international, page 87


Part 2 : 11/10/17 09:52:04

Question 101 - ICMA 10.P2.029 - Ratios: Liquidity, Leverage, Coverage and Activity

A. The price earnings ratio is a publicly-traded company's stock is its price per common share divided by its earnings
per common share. Purchasing inventory for cash will have no effect on either of those values or on the ratio.

B. Working capital is current assets minus current liabilities. Purchasing inventory with cash will not change current
assets, because cash will decrease and inventory will increase by the same amounts. Since current liabilities are not
affected by the purchase either, working capital will not change.

C. The quick or acid test ratio is (Cash + Marketable Securities + Accounts Receivable) ÷ Current Liabilities.
Inventory is not included in the numerator of the ratio. Purchasing inventory with cash will decrease cash and
decrease the numerator of the ratio, while the denominator will not change. The result will be a reduction in
the quick ratio, which is undesirable and thus an adverse impact.

D. The current ratio is current assets divided by current liabilities. Purchasing inventory with cash will not change
current assets, because cash will decrease and inventory will increase by the same amounts. Since current liabilities
are not affected by the purchase either, the current ratio will not change.

Question 102 - CMA 690 4.14 - Ratios: Liquidity, Leverage, Coverage and Activity

A. Accounts receivable turnover is calculated as annual credit sales divided by the average accounts
receivable. It measures the number of times the accounts receivable "turn over" during a year's time. If the
company lengthens the period for cash discounts, more customers will take longer to pay their bills, which
will increase the average receivables. This will, in turn, decrease the accounts receivable turnover ratio. A
decrease in the accounts receivable turnover ratio means the accounts turn over less frequently; and in this
case, that is because the level of accounts receivable is higher.

B. Accounts receivable turnover is calculated as annual credit sales divided by the average accounts receivable.
Therefore, this number will decrease if there is a decrease in credit sales or an increase in the average receivables.
Under the allowance for doubtful accounts method, the writing off of a receivable will not effect the accounts receivable
turnover, because the net average accounts receivable is used in the accounts receivable turnover ratio (i.e., net of the
allowance for doubtful accounts). The net accounts receivable balance does not change when an account is written
off. Accounts receivable decreases, but the negative (credit) balance in the allowance for doubtful accounts also
decreases. The effect on the combination of the two account balances is zero.

C. Accounts receivable turnover is calculated as annual credit sales divided by the average accounts receivable.
Therefore, this number will decrease if there is a decrease in credit sales or an increase in the average receivables. An
increase in cash sales in proportion to credit sales may or may not cause a decrease in the accounts receivable
turnover ratio. It would depend upon the magnitude of the change in both annual credit sales and average accounts
receivable.

D.

Accounts receivable turnover is calculated as annual credit sales divided by the average accounts receivable. The A/R
turnover will decrease if there is a decrease in credit sales or an increase in the average receivables or if both
decrease but the decrease in credit sales is proportionally greater than the decrease in average receivables.

A decrease in credit sales at the end of the period will decrease both the credit sales and the receivables balance at
the end of the period. Because annual credit sales is a larger amount than average accounts receivable, the amount of
decrease in sales will not be proportionally greater than the amount of the decrease in average accounts receivable.
Therefore, the accounts receivable turnover will not normally decrease as a result of a significant sales volume
decrease near the end of the accounting period.

(c) HOCK international, page 88


Part 2 : 11/10/17 09:52:04

Question 103 - CMA 1289 P4 Q13 - Ratios: Liquidity, Leverage, Coverage and Activity

A. Working capital is current assets minus current liabilities. In the process of paying and declaring dividends, working
capital is changed only by the declaration of the dividend. At that date working capital decreases and at all other dates
it is unchanged.

B. Working capital is current assets minus current liabilities. In the process of paying and declaring dividends, working
capital is changed by the declaration of the dividend. At that date working capital decreases and at all other dates it is
unchanged.

C. Working capital is current assets minus current liabilities. In the process of paying and declaring dividends, working
capital is changed by the declaration of the dividend. At that date working capital decreases and at all other dates it is
unchanged.

D. Working capital is current assets minus current liabilities. In the process of declaring and paying a
dividend, the working capital of the company decreases at the declaration date and is unchanged at all other
dates. The decrease at declaration occurs because the liabilities of the company increase with the dividend
payable while all other assets and liabilities remain the same. When the dividend is paid, both liabilities
(dividends payable) and assets (cash) are decreased by the same amount, which leaves working capital
unchanged.

Question 104 - ICMA 10.P2.043 - Ratios: Liquidity, Leverage, Coverage and Activity

A.

Financial leverage does not affect either EPS or EBIT. Financial leverage is a measure of the proportionate amount of
change in net income or EPS that results from a change in EBIT.

Operating leverage does not affect EBIT. Operating leverage is a measure of the proportionate amount of change in
EBIT that results from a change in Sales.

B.

The amount of variability in EBIT experienced by a firm is a function of its degree of operating leverage only. Therefore,
it is not true that both firms will have the same variability in EBIT. If Firm A has a higher degree of operating leverage
than Firm B, Firm A will have a higher variability in EBIT than Firm B will have.

However, if, for example, Firm A's DOL is 3.0 and its DFL is 1.5 and Firm B's DOL is 1.5 and its DFL is 3, both firms
will have the same amount of variability in EBT. This is true because DOL × DFL = DTL (Degree of Total Leverage).
Both firms' degree of total leverage will be 4.5, and an increase in sales of 10% for both firms will result in an increase
in EBT of 45% for both firms (10% × 4.5). Here is an example:
Firm A Firm B
After (sales After (sales
Before increase 10%) Before increase 10%)
Sales $1,000,000 $1,100,000 $1,000,000 $1,100,000
Variable Costs 250,000 275,000 550,000 605,000
Contribution Margin 750,000 825,000 450,000 495,000
Fixed Costs 500,000 500,000 150,000 150,000
EBIT 250,000 325,000 300,000 345,000
Interest expense 83,333 83,333 200,000 200,000
EBT $ 166,667 $ 241,667 $ 100,000 $ 145,000

Degree of Operating Leverage 3.0 1.5


Degree of Financial Leverage 1.5 3.0

(c) HOCK international, page 89


Part 2 : 11/10/17 09:52:04

Degree of Total Leverage 4.5 4.5

Percentage Increase in EBIT 30% 15%


Percentage Increase in EBT 45% 45%

C. A decrease in the financial leverage of a firm means the firm is using less debt in its capital structure. That would
usually cause a decrease in the beta value of the firm, because investors would perceive less investment risk in the
company's stock.

D.

When financial leverage is being used, an increase in Earnings Before Interest and Taxes (EBIT) will cause an
even greater increase in net income, and a decrease in EBIT will cause an even greater decrease in net
income.

EPS is simply net income per share, so we can say the same thing about EPS as we say about net income,
although we do need to change it a little because EBIT is for the company as a whole, whereas EPS is a per
share amount. When financial leverage is being used, an increase in EBIT will cause an even greater
proportionate increase in EPS, and a decrease in EBIT will cause an even greater proportionate decrease in
EPS.

Question 105 - ICMA 10.P2.048 - Ratios: Liquidity, Leverage, Coverage and Activity

A.

The debt-to-equity ratio is total liabilities divided by total equity. Using the information given, the first step is
to figure out what total liabilities are and what total equity is. Then, we can calculate the debt-to-equity ratio.

Current assets equal $640,000 and the current ratio is 3.2. Since the current ratio is current assets divided by
current liabilities, we can use that information to calculate what current liabilities are. When we know what
current liabilities are, we can add current liabilities to long-term liabilities, which is given, and we will have
total liabilities. Once we know what total liabilities are, we can subtract that from total assets and we will have
total equity.

Let X = current liabilities


Current ratio = current assets / current liabilities
3.2 = 640,000 / X
X = 200,000

Total liabilities = current liabilities + long-term liabilities


Total liabilities = 200,000 + 130,000
Total liabilities = 330,000

Total equity = total assets – total liabilities


Total equity = 990,000 – 330,000
Total equity = 660,000

Debt-to-equity ratio = total liabilities / total equity


Debt-to-equity ratio = 330,000 / 660,000
Debt-to-equity ratio = 0.50, or 0.50 to 1

B. This is long-term liabilities divided by total assets. The debt-to-equity ratio is total liabilities divided by total equity,
neither of which are given. They must be calculated using the information that is given before the debt-to-equity ratio
can be calculated.

(c) HOCK international, page 90


Part 2 : 11/10/17 09:52:04

C. This is total liabilities divided by total assets. The debt-to-equity ratio is total liabilities divided by total equity, neither
of which are given. They must both be calculated using the information that is given before the debt-to-equity ratio can
be calculated.

D. This is long-term liabilities divided by long-term assets. The debt-to-equity ratio is total liabilities divided by total
equity, neither of which are given. They must be calculated using the information that is given before the debt-to-equity
ratio can be calculated.

(c) HOCK international, page 91

You might also like